Quiz 1

¡Supera tus tareas y exámenes ahora con Quizwiz!

A 75-year-old woman is admitted to the hospital with anginal pain. ECG reveals myocardial infarction and a right bundle branch block. During physical examination the patient has a loud second heart sound. Which of the following heart valves are responsible for the production of the second heart sound? A. Aortic and pulmonary B. Aortic and tricuspid C. Tricuspid and mitral D. Mitral and pulmonary E. Tricuspid and pulmonary

A. Aortic and pulmonary The second heart sound is caused by the closure of the aortic and pulmonary valves.

A 33-year-old male is admitted to the hospital after a violent, multiple car collision. His blood pressure is 89/39 mm Hg, and a central venous line is ordered to be placed. Which of the following structures is used as a landmark to place the tip of the catheter of the central venous line? A. Carina B. Subclavian artery C. Superior vena cava D. Left atrium E. Right atrium

A. Carina The carina is the only answer listed that can easily be seen in radiograph. The carina is at the level of T4-5 (plane associated with the sternal angle of Louis). This landmark is commonly used to guide the placement of a central venous line.

A 30-year-old man is admitted to the emergency department because of significant nosebleeding and a headache that has worsened over several days. He also complains of fatigue. Upon examination it is noted that brachial artery pressure is markedly increased, femoral pressure is decreased, and the femoral pulses are delayed. The patient shows no external signs of inflammation. Which of the following is the most likely diagnosis? A. Coarctation of the aorta B. Cor pulmonale C. Dissecting aneurysm of the right common iliac artery D. Obstruction of the superior vena cava E. Pulmonary embolism

A. Coarctation of the aorta Increased arterial pressure in the upper limbs (as demonstrated in the brachial artery) and decreased pressure in the lower limbs (as demonstrated in the femoral artery) are common symptoms of coarctation of the aorta. Other symptoms include tortuous and enlarged blood vessels above the coarctation and an increased risk of cerebral hemorrhage. This condition of coarctation occurs when the aorta is abnormally constricted during development.

A 28-year-old woman in the third trimester of pregnancy has experienced severe dizziness for several days and is admitted to the hospital. During physical examination her blood pressure is normal when standing or sitting. When the patient is supine, her blood pressure drops to 90/50 mm Hg. What is the most likely explanation for these findings? A. Compression of the inferior vena cava B. Compression of the superior vena cava C. Compression of the aorta D. Compression of the common carotid artery E. Compression of the internal jugular veins

A. Compression of the inferior vena cava The inferior vena cava quite likely undergoes compression by the growing fetus when the mother is in the supine position. In this case the compression led to reduced blood flow through the heart, with a resultant drop in blood pressure.

A 4-year-old male is operated on for a correction of a small, muscular interventricular septal defect. To access the right side of the intraventricular septum, a wide incision is first made in the anterior surface of the right atrium. Instruments are then inserted through the tricuspid valve to correct the ventricular septal defect. Which of the following structures is the most crucial to protect during the opening of the right atrium? A. Crista terminalis B. Pectinate muscles C. Tricuspid valve D. Eustachian valve E. Coronary sinus

A. Crista terminalis The crista terminalis is a muscular ridge that runs from the opening of the superior vena cava to the inferior vena cava. This ridge provides the path taken by the posterior internodal pathway (of Thorel) between the sinoatrial and atrioventricular nodes. The crista also provides the origin of the pectinate muscles of the right auricle.

A 41-year-old female is admitted to the emergency department with a complaint of severe, sharp, but poorly localized pain on the chest wall. Radiographic examination gives evidence of pleural effusion. What is the location of the neuronal cell bodies responsible for the nerve fibers that carry this pain to the central nervous system (CNS)? A. Dorsal root ganglia B. Sympathetic chain ganglia C. Dorsal horn of the spinal cord D. Lateral horn of the spinal cord E. Ventral horn of the spinal cord

A. Dorsal Root Ganglion The dorsal root ganglia contain nerve cell bodies for general somatic afferent and general visceral afferent neuronal processes. Pain localized on the chest wall is transmitted back to the CNS via sensory fibers.

A 25-year-old female is admitted to the hospital after a violent automobile crash. Radiographic examination reveals four broken ribs in the left thoracic wall, producing a flail chest observable on physical examination. Which of the following conditions is most likely to also be observed during physical examination? A. During deep inspiration the flail segment moves in the opposite direction of the chest wall. B. During deep inspiration the flail segment moves in the same direction as the chest wall. C. "Pump handle movements" of the ribs will not be affected by the rib fractures. D. The descent of the diaphragm will be affected on the side of the broken ribs. E. The descent of the diaphragm will be affected on the side of the broken ribs and also on the opposite side.

A. During deep inspiration the flail segment moves in the opposite direction of the chest wall When multiple rib fractures produce a flail segment of the thoracic wall, paradoxical motion of the flail segment is commonly experienced upon deep inspiration; that is, the flail area is sucked in rather than expanding outward with inspiration, and the reverse movement occurs in expiration.

A 55-year-old woman is admitted to the hospital with cough and severe dyspnea. Radiographic examination reveals that the patient suffers from emphysema. Upon physical examination the patient shows only "bucket handle movements" during deep inspiration. Which of the following movements of the thoracic wall is characteristic for this type of breathing? A. Increase of the transverse diameter of the thorax B. Increase of the anteroposterior diameter of the thorax C. Increase of the vertical dimension of the thorax D. Decrease of the anteroposterior diameter of the thorax E. Decrease of the transverse diameter of the thorax

A. Increase of the transverse diameter of the thorax The "bucket handle movement" of the ribs affects the transverse diameter of the thorax. Inspiration would increase the transverse diameter, whereas expiration decreases the transverse diameter.

A 42-year-old man is admitted to the hospital with retrosternal pain. Endoscopy and biopsy examina- tions of the trachea reveal a malignant growth at the right main bronchus. Which of the following lymph nodes will most likely be the first infiltrated by cancer- ous cells from the malignancy? A. Inferior tracheobronchial B. Paratracheal C. Bronchomediastinal trunk D. Bronchopulmonary E. Thoracic duct

A. Inferior tracheobronchial Lymph from the right primary bronchus would drain fi rst into the inferior tracheobronchial nodes.

A 47-year-old woman is admitted to the hospital with pain in her neck. During physical examination it is observed that the thyroid gland is enlarged and is compressing the trachea. A biopsy reveals a benign tumor. A CT scan examination reveals tracheal deviation to the left. Which of the following structures will most likely be compressed as a result of the deviation? A. Left brachiocephalic vein B. Left internal jugular vein C. Left subclavian artery D. Vagus nerve E. Phrenic nerve

A. Left brachiocephalic vein A left tracheal deviation with an enlarged thyroid gland will most likely compress the left brachiocephalic vein.

An unconscious 2-month-old infant is admitted to the emergency department after an automobile collision. An emergency tracheostomy is performed. Which of the following structures is most commonly at high risk of injury during this procedure? A. Left brachiocephalic vein B. Left common carotid artery C. Vagus nerve D. Phrenic nerve E. Thoracic duct

A. Left brachiocephalic vein In a tracheotomy, an incision is made at the level of the sixth cervical vertebra, near the cricoid cartilage. The left brachiocephalic vein passes across the trachea immediately anterior to the brachiocephalic trunk. This vein is the most superficial structure and thus the most likely to be damaged.

A 48-year-old male patient is scheduled to have a coronary arterial bypass because of chronic angina. Coronary arteriography reveals nearly total blockage of the posterior descending interventricular artery. In exposing this artery to perform the bypass procedure, which ac- companying vessel is most susceptible to injury? A. Middle cardiac vein B. Great cardiac vein C. Small cardiac vein D. Anterior cardiac vein E. Coronary sinus

A. Middle Cardiac vein The middle cardiac veins run parallel with the posterior interventricular (posterior descending) artery and drains directly into the coronary sinus.

A 35-year-old woman is admitted to the hospital with dyspnea. During physical examination her S 1 heart sound is very loud. Which of the following valves is most likely defective? A. Mitral valve B. Aortic C. Pulmonary D. Aortic and pulmonary E. Tricuspid

A. Mitral valve The mitral valve corresponds to the S1 heart sound produced during systole.

A 68-year-old male patient in the cardiology ward complains at each mealtime of difficulty in swallowing (dysphagia). Radiographic studies reveal significant cardiac hypertrophy. A barium swallow, followed by radiographic examination of the thorax, reveals esophageal constriction directly posterior to the heart. Which of the following is the most likely cause of the patient's dysphagia? A. Mitral valve stenosis B. Pulmonary valve stenosis C. Regurgitation of the aorta D. Occlusion of the anterior interventricular artery E. Occlusion of the posterior interventricular artery

A. Mitral valve stenosis Mitral stenosis leads to left atrial dilation, which can exert a compressive effect on the esophagus.

A 33-year-old male is admitted to the hospital with severe traumatic injuries. His blood pressure is 89/39 mm Hg, and a central venous line is ordered to be placed. Which of the following injuries is most likely to occur when a subclavian central venous line procedure is performed? A. Penetration of the subclavian artery B. Impalement of the phrenic nerve C. Penetration of the superior vena cava D. Penetration of the left common carotid artery E. Impalement of the vagus nerve

A. Penetration of the subclavian artery The subclavian artery lies directly posterior to the subclavian vein; therefore, it is the structure that would be most vulnerable to damage when placing a central venous line in the subclavian vein.

A 62-year-old female is admitted to the hospital with severe dyspnea and also complains of pain over her left shoulder. A radiographic examination reveals an aneurysm of the aortic arch. Which of the following nerves is most likely affected by the aneurysm? A. Phrenic B. Vagus C. Cardiopulmonary D. Intercostal E. Thoracic splanchnic

A. Phrenic An aneurysm of the aortic arch could impinge upon the phrenic nerve, causing referral of pain to the left shoulder. This referral occurs because the root levels of the phrenic nerve are C3 to C5, nerve levels that are also distributed to the skin over the shoulder region.

A 39-year-old man is admitted to the hospital with odynophagia. A barium swallow reveals an esophageal constriction at the level of the diaphragm. A CT scan and a biopsy further indicate the presence of an esophageal cancer. Which of the following lymph nodes will most likely be affected first? A. Posterior mediastinal and left gastric B. Bronchopulmonary C. Tracheobronchial D. Inferior tracheobronchial E. Superior tracheobronchial

A. Posterior mediastinal and left gastric Lymph from the lower third of the esophagus drains into the posterior mediastinal and left gastric lymph nodes.

A 42-year-old male was admitted to the hospital after a head-on vehicular collision in which he received severe blunt trauma to his sternum from the steering wheel. What part of the heart would be most likely to be injured by the impact? A. Right ventricle B. Apex of left ventricle C. Left ventricle D. Right atrium E. Anterior margin of the left atrium

A. Right Ventricle These components of the heart are readily viewed in a plain radiograph of the thorax. It is important to understand the spatial arrangement of the heart as it rests in the thorax. The conus region of the right ventricle is located on the most anterior aspect of the heart, thus it is the most anterior portion of the heart within the thorax.

A 45-year-old woman is admitted to the hospital with severe dyspnea. Radiographic examination confirms the presence of a Pancoast tumor. Physical examination reveals that the patient has miosis of the pupil, partial ptosis of the eyelid, and anhydrosis of the face. Which of the following structures has most likely been injured? A. Sympathetic chain B. Vagus nerve C. Phrenic nerve D. Arch of aorta E. Cardiopulmonary plexus

A. Sympathetic chain Miosis, partial ptosis, and anhydrosis are a clinically important constellation of symptoms possibly indicating Horner syndrome. Horner syndrome is a lesion of the cervical sympathetic chain and sympathetic chain ganglia and is often a result of a Pancoast tumor, also known as a superior pulmonary sulcus tumor of the apex of the lung. The pupil, eyelid (superior tarsal muscle), and sweat glands are all under sympathetic nervous system control.

A 54-year-old male is admitted to the hospital with dyspnea. Imaging and physical examination and echocardiographic studies reveal severe mitral valve prolapse. Auscultation of this valve is best performed at which location? A. Left fifth intercostal space, just below the nipple B. Right lower part of the body of the sternum C. Right second intercostal space near the lateral border of the sternum D. Directly over the middle of the manubrium E. Left second intercostal space near the lateral border of the sternum

A. The left fifth intercostal space, just below the nipple The left fifth intercostal space, just below the left nipple, is typically the location to listen to the mitral valve.

A 5-year-old boy had been playing with his little race cars. Soon after he put a wheel from one of the cars in his mouth, he began choking and coughing. Where in the tracheobronchial tree is the most common site for a foreign object to lodge? A. The right primary bronchus B. The left primary bronchus C. The carina of the trachea D. The beginning of the trachea E. The left tertiary bronchus

A. The right primary bronchus The right main bronchus is the shorter, wider, and more vertical primary bronchus. Therefore, this is most often the location that foreign objects will likely be lodged.

A 35-year-old female who was brought into the emergency department for a drug overdose requires insertion of a nasogastric tube and administration of activated charcoal. What are the three sites in the esophagus where one should anticipate resistance due to compression on the organ? A. At the aortic arch, the cricopharyngeal con- striction, and the diaphragmatic constriction B. The cardiac constriction, the cricoid cartilage constriction, and the thoracic duct C. The pulmonary constriction, cricothyroid con- striction, and the azygos arch D. The cardiac constriction, the azygos arch, the pulmonary trunk E. The cricopharyngeal constriction, cricothyroid constriction, thymus gland

A. at the aortic arch, the cricopharyngeal constriction, and the diaphragmatic constriction The esophagus typically has four constrictions. In the thorax the esophagus is compressed by (1) the arch of the aorta, (2) the left principal bronchus, and (3) the diaphragm. The cricopharyngeal constriction is in the neck.

A 35-year-old man is admitted to the hospital with pain on swallowing. Imaging reveals a dilated left atrium. Which structure is most likely being compressed by the expansion of the left atrium to result in the patient's symptoms? A. Esophagus B. Root of the lung C. Trachea D. Superior vena cava E. Inferior vena cava

A. esophagus The patient's chief complaint is pain upon swallowing. With a dilated left atrium, the most probable structure being compressed is the esophagus. The esophagus descends into the abdomen immediately posterior to the left atrium below the level of the tracheal carina.

A 45-year-old female is admitted to the hospital with difficulty breathing. Radiographic examination reveals a tumor invading the lung surface anterior to the hilum. Which nerve is most likely compressed by the tumor to result in dyspnea? A. Phrenic B. Vagus C. Intercostal D. Recurrent laryngeal E. Cardiopulmonary

A. phrenic nerve the phrenic nerve runs anterior to the root of the lung and innervates the diaphragm. path between the anterior medial aspect of the lung and the mediastinum.

A 35-year-old man is admitted to the hospital with severe chest pain, dyspnea, tachycardia, cough, and fever. Radiographic examination reveals significant pericardial effusion. When pericardiocentesis is per- formed, the needle is inserted up from the infrasternal angle. The needle passes too deeply, piercing the vis- ceral pericardium and entering the heart. Which of the following chambers would be the first to be penetrated by the needle? A. Right ventricle B. Left ventricle C. Right atrium D. Left atrium E. The left cardiac apex

A. right ventricle Pericardiocentesis is usually performed through the infrasternal angle with the needle passing up through the diaphragm to the fi brous pericardium. The diaphragmatic surface of the heart is largely composed of the right ventricle and would therefore be entered if a needle is inserted too far.

A 42-year-old woman is admitted to the hospital after blunt trauma to her sternum by the steering wheel during a car crash. Radiographic examination reveals a cardiac tamponade. ECG data indicate that the heart has been severely injured. Which of the following cardiac structures will most likely be injured? A. Right ventricle B. Obtuse margin of the left ventricle C. Right atrium D. Left atrium E. Apex of the left ventricle

A. right ventricle The sternocostal surface of the heart consists mostly of the right ventricle. Therefore, an anterior injury to the thorax would mostly likely first affect the right ventricle because it is adjacent to the deep surface of the sternum.

A 34-year-old male with a complaint of sharp, localized pain over the thoracic wall is diagnosed with pleural effusion. A chest tube is inserted to drain the effusion through an intercostal space. At which of the following locations is the chest tube most likely to be inserted? A. Superior to the upper border of the rib B. Inferior to the lower border of the rib C. At the middle of the intercostal space D. Between the internal and external intercostal muscles E. Between the intercostal muscles and the posterior intercostal membrane

A. superior to the upper border of the rib The location where one is least likely to damage important structures by making an incision or pushing a chest tube into the thorax is over the upper border of the rib.

A 22-year-old woman had undergone elective breast enhancement, with the insertion of 250-ml saline bags bilaterally. This resulted, unfortunately, in loss of sensation bilaterally in the nipples and areolae and some reduction of sensation of the skin from the areolae laterally to the midaxillary lines. Which of the following nerves were most likely subject to iatrogenic injury? A. Anterior cutaneous branches of second and third intercostal nerves B. Anterior and lateral cutaneous branches of the fourth intercostal nerves C. Lateral pectoral nerves D. Cutaneous branches of the second thoracic spinal nerves (intercostobrachial nerves) E. Lateral cutaneous branches of the second and third intercostal nerves

B. Anterior and Lateral cutaneous branches of the fourth intercostal nerves The anterior and lateral cutaneous branches of the fourth intercostal nerves provide the sensory and sympathetic supply to the areolae and nipples.

A 55-year-old patient is to undergo a coronary bypass operation. The artery of primary concern is the vessel that supplies much of the left ventricle and the right and left bundle branches of the cardiac conduction system. Which artery is the surgeon most concerned with? A. Right marginal B. Anterior interventricular C. Circumflex D. Artery to the sinoatrial (SA) node E. Posterior interventricular

B. Anterior interventricular The anterior interventricular artery supplies the right and left ventricles and anterior two thirds of the IV septum.

A 72-year-old patient vomited and then aspirated some of the vomitus while under anesthesia. On bronchoscopic examination, partially digested food is observed blocking the origin of the right superior lobar bronchus. Which of the following groups of bronchopulmonary segments will be affected by this obstruction? A. Superior, medial, lateral, medial basal B. Apical, anterior, posterior C. Posterior, anterior, superior, lateral D. Apical, lateral, medial, lateral basal E. Anterior, superior, medial, lateral

B. Apical, anterior, posterior The superior lobar bronchus is one of the divisions of the right main bronchus. This bronchus branches into apical, anterior, and posterior tertiary bronchi.

A 42-year-old woman is seen by her family physician because she has a painful lump in her right breast and a bloody discharge from her right nipple. Upon physical examination it is noted that there is unilateral inversion of the right nipple and a hard, woody texture of the skin over a mass of tissue in the right upper quadrant of the breast. Which of the following conditions is most frequently characterized by these symptoms? A. Peau d'orange B. Cancer en cuirasse C. Intraductal cancerous tumor D. Obstruction of the lymphatics draining the skin of the breast, with edema of the skin E. Inflammation of the epithelial lining of the nipple and underlying hypodermis

B. Cancer en cuirasse All of the symptoms described in the question are indicative of breast cancer. The best choice of answers is cancer en cuirasse, a pathologic condition that presents as a hard, "woodlike" texture.

A 42-year-old woman is admitted to the emergency department after a fall from the balcony of her apartment. During physical examination there is an absence of heart sounds, reduced systolic pressure, reduced cardiac output, and engorged jugular veins. Which condition is most likely characterized by these signs? A. Hemothorax B. Cardiac tamponade C. Hemopneumothorax D. Pneumothorax E. Deep vein thrombosis

B. Cardiac Tamponade Cardiac tamponade is a condition in which fluid accumulates in the pericardial cavity. It can result from pericardial effusion or from leakage of blood from the heart or proximal portions of the great vessels. The increased pressure within the pericardial sac leads to decreased cardiac filling during diastole and therefore reduced systolic blood pressure. Because of the reduced pumping capacity of the heart, there is increased pressure in the venous system, leading to the distension of the jugular venous system.

A 75-year-old man is scheduled for his routine annual medical examination. During echocardiographic examination a large, mobile structure resembling a thrombus is identified in the right atrium near the opening of the inferior vena cava. After careful examination the doctor identifies the large mobile structure as a normal component of the heart. Which of the following structures could most likely resemble a thrombus in this location? A. Tricuspid valve B. Eustachian valve C. Thebesian valve D. Septum primum E. Fossa ovalis

B. Eustachian valve The eustachian valve is an embryologic remnant of the valve of the inferior vena cava and is not a functional valve.

A 3-year-old male who fell from a tree complains of severe pain over the right side of his chest because of a rib fracture at the midaxillary line. He is admitted to the hospital due to his difficulty breathing. Radiographic and physical examinations reveal atelectasis, resulting from the accumulation of blood in his pleural space and resulting hemothorax. What is the most likely the source of bleeding to cause the hemothorax? A. Left common carotid artery B. Intercostal vessels C. Pulmonary arteries D. Pulmonary veins E. Internal thoracic artery

B. Intercostal vessels Due to rib fracture, the intercostal vessels are damaged, parietal pleura is torn, and blood flows into the pleural space. The loss of negative pressure within the pleural cavity results in collapse of the lung.

A 52-year-old patient is admitted to the hospital with severe chest pain. ECG and radiographic examinations provide evidence of a significant myocardial infarction and cardiac tamponade. An emergency pericardiocentesis is ordered. At which of the following locations will the needle best be inserted to relieve the tamponade? A. Right seventh intercostal space in the midaxillary line B. Left fifth intercostal space in the midclavicular line C. Right third intercostal space, 1 inch lateral to the sternum D. Left sixth intercostal space, 1 ⁄ 2 inch lateral to the sternum E. Triangle of auscultation

B. Left fifth intercostal space in the midclavicular line During pericardiocentesis, the needle is inserted below the xiphoid process, or in the left fifth intercostal space in the midclavicular line. The most effective way of draining the pericardium is by penetrating the thoracic wall at its lowest point anatomically

A 39-year-old male is admitted to the hospital with a complaint of severe retrosternal pain that radiates to the left shoulder. The pain is relieved by leaning forward. Auscultation reveals a pericardial friction rub, leading to a diagnosis of pericarditis. Which of the following nerves is responsible for the radiating pain? A. Intercostobrachial B. Phrenic C. Long thoracic D. Greater thoracic splanchnic E. Cardiopulmonary

B. Phrenic Pericarditis is an inflammation of the pericardium and often causes a pericardial friction rub, with the surface of the pericardium becoming gradually coarser. Because the phrenic nerve is solely responsible for innervation of the pericardium, it would transmit the pain fibers radiating from the pericardial friction rub. The phrenic nerve contains sensory nerve fibers from C3 to C5, spinal nerve levels that also supply the skin of the shoulder area; therefore, pain carried by the phrenic nerve may be referred to the shoulder.

A 35-year-old woman is admitted to the hospital with a complaint of shortness of breath. During physical examination it is noted that there is wide splitting in her S 2 heart sound. ECG reveals a right bundle branch block. Which of the following valves is most likely defective? A. Mitral valve B. Pulmonary C. Aortic and mitral D. Tricuspid E. Tricuspid and aortic

B. Pulmonary The pulmonary valve is associated with the S 2 heart sound produced in diastole. A splitting in the S 2 sound indicates that the aortic and pulmonary valves are not closing simultaneously and would correlate with a possible defect in this valve.

A 35-year-old woman is admitted to the hospital with a complaint of shortness of breath. During physical examination it is noted that there is wide splitting in her S 2 heart sound. Which of the following valves is/are responsible for production of the S 2 heart sound? A. Mitral valve B. Pulmonary and aortic C. Aortic and mitral D. Tricuspid E. Tricuspid and aortic

B. Pulmonary and aortic The S 2 heart sound refers to the second (dub) heart sound. This sound is produced by the closure of the aortic and pulmonary semilunar valves.

A 62-year-old female is admitted to the emergency department with severe chest pains that radiate to her left arm. ECG reveals that the patient suffers from an acute myocardial infarction. Coronary angiography is performed and a stent is placed at the proximal portion of the anterior interventricular artery (left anterior descending). Because of the low ejection fraction of the right and left ventricles, a cardiac pace- maker is also placed in the heart. The function of which of the following structures is essentially replaced by the insertion of a pacemaker? A. AV node B. SA node C. Purkinje fibers D. Bundle of His E. Bundle of Kent

B. SA node The SA node functions as the primary intrinsic pacemaker of the heart, setting the cardiac rhythm. An artificial pacemaker assists in producing a normal rhythm when the SA node is not functioning normally.

A 22-year-old woman sustained a chest injury upon impact with the steering wheel during a car crash. Upon admission of the patient to the hospital, physical examination revealed profuse swelling, inflammation, and deformation of the chest wall. A radiograph revealed an uncommon fracture of the manubrium at the sternomanubrial joint. Which of the following ribs would be most likely to also be involved in such an injury? A. First B. Second C. Third D. Fourth E. Fifth

B. Second The superior margin of the manubrium is char- acterized by the jugular notch. Laterally are the sternoclavicular joints and the articulations of the first ribs with the manubrium. The second pair of ribs articulates with the sternum at the sternal angle, the junction of the manubrium with the body of the sternum.

A 55-year-old male is undergoing an aortic valve replacement. During the procedure the heart is connected to the heart lung machine. As the surgeon explores the oblique pericardial sinus, which of the following is not directly palpable with the tips of the fingers? A. Inferior vena cava B. Superior vena cava C. Posterior wall of the left atrium D. Inferior right pulmonary vein E. Right atrium

B. Superior vena cava The superior vena cava empties into the right atrium on the superior aspect of the heart; it is not directly palpable from the oblique sinus. The oblique sinus is a blind cul-de-sac providing access to the inferior vena cava, the posterior wall of the left atrium, right atrium, and the right and left pulmonary veins.

A 45-year-old man is admitted to the hospital with severe chest pain radiating to his left arm and left upper jaw. An emergency ECG reveals an acute myocardial infarction of the posterior left ventricular wall. Which of the following spinal cord segments would most likely receive the sensations of pain in this case? A. T1, T2, T3 B. T1, T2, T3, T4 C. T1, T2 D. T4, T5, T6 E. T5, T6, T7

B. T1, T2, T3, T4 The pain experienced by the patient travels with the sympathetic innervation of the heart, derived from spinal nerve levels T1 to T4. The pain fibers leave the heart and the cardiac plexuses via the cardiopulmonary nerves. Subsequently, the pain fibers pass through the sympathetic chain, enter the spinal nerve, and pass into the dorsal roots of the spinal nerves. The cell bodies of the pain fibers are located in the dorsal root ganglia of the spinal nerves from T1 to T4.

A 51-year-old female with a history of brain tumor and associated severe oropharyngeal dysphagia develops right lower lobe pneumonia after an episode of vomiting. Which of the following is the best reason that this type of aspiration pneumonia most commonly affects the right lower lung lobe? A. Pulmonary vascular resistance is higher in the right lung than the left lung. B. The right main bronchus is straighter than the left main bronchus. C. The right main bronchus is narrower than the main bronchus. D. The right main bronchus is longer than the left main bronchus. E. The right lower lung lobe has poorer venous drainage than the other lobes.

B. The right main bronchus is straighter than the left main bronchus The right primary bronchus is shorter, wider, and more vertical than the left main bronchus. When a foreign body is aspirated, it is more likely to enter the right main bronchus (although in some cases the foreign body enters the left bronchus).

A 47-year-old male is admitted to the emergency department, due to severe dysphagia. Edema of the lower limbs is apparent upon physical examination. A barium sulfate swallow imaging procedure reveals esophageal dilation, with severe inflammation, due to constriction at the esophageal hiatus. What is the most likely cause of the severe edema of the lower limbs? A. Thoracic aorta constriction B. Thoracic duct blockage C. Superior vena caval occlusion D. Aortic aneurysm E. Femoral artery disease

B. Thoracic duct blockage The thoracic duct is important in lymph drainage of the entire body with the exception of the upper right quadrant. The thoracic duct ascends between the aorta and azygos vein behind the esophagus. Dilation of the esophagus here in the lower thorax can compress the thoracic duct, leading to impairment of lymphatic drainage and resultant edema.

A 48-year-old male patient is admitted with chronic angina. Coronary angiography reveals nearly total blockage of the circumflex artery near its origin from the left coronary artery. When this artery is exposed to perform a bypass procedure, what accompanying vein must be protected from injury? A. Middle cardiac B. Great cardiac C. Small cardiac D. Anterior cardiac E. Posterior cardiac

B. great cardiac The great cardiac vein (anterior interventricular vein) takes a pathway initially beside the anterior interventricular coronary artery (left anterior descending: LAD) in its course, finally terminating in the coronary sinus when it is joined by the oblique vein of the left atrium (of Marshall).

A 35-year-old male bartender is admitted to the hospital due to severe dysphagia. A CT scan ( Fig. 2-3 ) reveals carcinoma of the middle segment of the esophagus. Which of the following structures will most likely be affected if the carcinoma increases greatly in size? A. Inferior vena cava B. Left atrium C. Pulmonary artery D. Left ventricle E. Vertebral body

B. left atrium The esophagus lies posterior to the heart. Of the four chambers in the heart, the left atrium lies most posteriorly, just anterior to the esophagus when the heart is in its normal position in the mediastinum.

A 34-year-old male unconscious patient is admitted to the hospital. His blood pressure is 85/45 mm Hg. A central venous line is ordered to be placed. During subsequent radiographic examination a chylothorax is detected. Which of the following structures was most likely accidentally damaged during the placement of the central venous line? A. Left external jugular vein B. Site of origin of the left brachiocephalic vein C. Right subclavian vein D. Proximal part of right brachiocephalic vein E. Right external jugular vein

B. site of origin of the left brachiocephalic vein Chylothorax is usually caused by injury to the thoracic duct. The thoracic duct enters the venous system at the junction of the left internal jugular vein and the left subclavian vein, where they form the left brachiocephalic vein. Penetrating injuries at the beginning of the left brachiocephalic vein commonly also disrupt the termination of the thoracic duct.

A 54-year-old female is admitted to the hospital with a stab wound of the thoracic wall in the area of the right fourth costal cartilage. Which of the following pulmonary structures is present at this site? A. The horizontal fissure of the left lung B. The horizontal fissure of the right lung C. The oblique fissure of the left lung D. The apex of the right lung E. The root of the left lung

B. the horizontal fissure of the right lung The horizontal fissure of the right lung is a fissure separating the superior lobe from the middle lobe. It usually extends medially from the oblique fissure at the midaxillary line to the sternum, along the lower border of the fourth rib.

A 70-year-old male with a history of two previous myocardial infarctions is admitted to the hospital with severe chest pain. ECG reveals a new myocardial infarction and ventricular arrhythmia. Coronary angiography reveals that the right coronary artery is blocked just distal to the origin of the right marginal artery in a right coronary dominant circulation. Which of the following structures would most likely be affected after such a blockade? A. Right atrium B. SA node C. AV node D. Lateral wall of the left ventricle E. Anterior interventricular septum

C. AV node The atrioventricular (AV) node is most commonly supplied by a branch of the right coronary artery. This branch arises at the crux of the heart (the point of junction of all four cardiac chambers posteriorly); this is the location of the occlusion.

A 29-year-old patient complains of severe pain radiating across her back and chest. Upon clinical ex- amination you observe a rash characteristic of herpes zoster infection passing from her upper left back and across her left nipple. Which of the following spinal nerve roots sheds the active virus? A. Dorsal root of T3 B. Ventral root of T3 C. Dorsal root of T4 D. Ventral root of T4 E. Dorsal root of T5

C. Dorsal root of T4 The dermatome that encompasses the nipple is supplied by spinal nerve T4. In this case the herpes zoster virus is harbored in the dorsal root ganglion of T4 and can be activated to cause the characteristic rash that is distributed along the dermatome including the nipple.

A 34-year-old male with a complaint of sharp, localized pain over the thoracic wall is diagnosed with pleural effusion. Through which intercostal space along the midaxillary line is it most appropriate to insert a chest tube to drain the effusion fluid? A. Fourth B. Sixth C. Eighth D. Tenth E. Twelfth

C. Eighth To avoid damaging the lungs, a chest tube should be placed below the level of the lungs, in the costodiaphragmatic recess. Such a point of entrance for the tube would be the eighth or ninth intercostal space.

A 58-year-old woman is admitted to the emer- gency department with severe dyspnea. Bronchoscopy reveals that the carina is distorted and widened. En- largement of which group of lymph nodes is most likely responsible for altering the carina? A. Pulmonary B. Bronchopulmonary C. Inferior tracheobronchial D. Superior tracheobronchial E. Paratracheal

C. Inferior tracheobronchial The inferior tracheobronchial nodes are also known as the carinal nodes and are located on the inferior aspect of the carina, the site of bifurcation of the trachea.

Following the diagnosis of breast cancer, a 42- year-old woman underwent a total mastectomy, including excision of the axillary tail (of Spence). Postoperatively, the patient complains of dysesthesia in the inner aspect of the arm and axilla. Which of the following nerves was most likely injured during the procedure? A. Ulnar B. Long thoracic C. Intercostobrachial D. Lateral cutaneous nerve of T4 E. Axillary nerve

C. Intercostobrachial The intercostobrachial nerve is responsible for innervation of the skin on the medial surface of the arm.

A 55-year-old man is brought to the emergency department after his motorcycle collided with an automobile. He is hypotensive, his pulse is irregular, and he shows other signs of substantial blood loss. MRI and CT scan evaluations reveal profuse abdominal bleeding. A decision is made to enter the chest so that the descend- ing thoracic aorta can be clamped to minimize blood loss and to preserve cerebral blood flow. After surgical entrance into the thorax, the fibrous pericardium is elevated with a forceps and punctured. A midline, longitudinal incision of the pericardium would best be made to prevent injury to which of the following structures? A. Auricular appendage of the left atrium B. Coronary sinus C. Left anterior descending artery D. Left phrenic nerve E. Left sympathetic trunk

C. Left anterior descending artery The anterior interventricular (left anterior descending) artery lies anteriorly and to the left and descends vertically to the left toward the apex. It can be more easily injured by a transverse incision of the pericardium, which would cross perpendicular to this artery.

A 39-year-old woman visits the outpatient clinic and complains of inability to reach a pantry shelf just above her head. History reveals that 2 months ago she underwent a mastectomy procedure and she did not have this complaint prior to the surgery. Which nerve was most likely damaged during surgery to result in the patient's complaint? A. Axillary B. Spinal accessory C. Long thoracic D. Radial E. Thoracodorsal

C. Long Thoracic. During mastectomy procedures, three superficial nerves are susceptible to ligation or laceration: the long thoracic nerve, intercostobrachial nerve, and thoracodorsal nerve. In the event of injury to the long thoracic nerve, the patient complains of an inability to fully abduct the humerus above the horizontal. The serratus anterior (supplied by the long thoracic nerve) is necessary to elevate, rotate, and abduct the scapula, to facilitate abduction of the humerus above the shoulder.

A 17-year-old girl is admitted to the hospital with dyspnea and fever. Radiographic examination reveals lobar pneumonia in one of the lobes of her right lung. During stethoscope examination at the level of the sixth intercostal space at the midaxillary line, rales (or crackles) are heard and dull sounds are produced dur- ing percussion. Which of the following lobes is most likely to be involved by pneumonia? A. Upper lobe of the right lung B. Middle lobe of the right lung C. Lower lobe of the right lung D. Lower lobes of the right and left lungs E. Upper lobes of the right and left lungs

C. Lower lobe of the right lung Crackling noises in the lungs due to the buildup of fluid are referred to as rales. The fluid usually migrates to the inferior portion of the lung due to the effects of gravity. Auscultation over the sixth intercostal space at the midaxillary line would be associated with the lower lobe of the right lung. Remember that the oblique fissure runs from the level of T2 posteriorly to the sixth costal cartilage anteriorly.

A 51-year-old male is admitted to the hospital with severe dyspnea. Radiographic examination reveals a tension pneumothorax. Adequate local anesthesia of the chest wall prior to insertion of a chest tube is necessary for pain control. Of the following layers, which is the deepest that must be infiltrated with the local anesthetic to achieve adequate anesthesia? A. Endothoracic fascia B. Intercostal muscles C. Parietal pleura D. Subcutaneous fat E. Visceral pleura

C. Parietal pleura The parietal pleura is innervated by the intercostal nerves and is very sensitive to pain, in this case being somatic innervation. Therefore, the parietal pleura is the deepest layer that must be anesthetized to reduce pain during aspiration or chest tube placement.

In coronary bypass graft surgery of a 49-year-old female, the internal thoracic artery is used as the coronary artery bypass graft. The anterior intercostal arteries in intercostal spaces three to six are ligated. Which of the following arteries will be expected to supply these intercostal spaces? A. Musculophrenic B. Superior epigastric C. Posterior intercostal D. Lateral thoracic E. Thoracodorsal

C. Posterior intercostal The anterior intercostal arteries anastomose with the posterior intercostal arteries. Ligation of the anterior arteries would not affect the supply of the intercostal spaces because the posterior arteries would provide collateral arterial supply.

A 22-year-old marathon runner is admitted to the emergency department with severe dyspnea. Physical examination reveals that the patient is experiencing an acute asthma attack, and a bronchodilating drug is administered. Which of the following elements of the nervous system must be inhibited by the drug to achieve relaxation of the smooth muscle of the tracheobronchial tree? A. Postganglionic sympathetic fibers B. Preganglionic sympathetic fibers C. Postganglionic parasympathetic fibers D. Visceral afferent fibers E. Somatic efferent fibers

C. Postganglionic parasympathetic fibers Postganglionic parasympathetic fibers are involved in the constriction of smooth muscle in the tracheoesophageal tree.

A 55-year-old female visited her doctor because of a painful lump in her right breast and a bloody dis- charge from her right nipple. Radiographic studies and physical examination reveal unilateral inversion of the nipple, and a tumor in the right upper quadrant of the breast is suspected. In addition, there is an orange-peel appearance of the skin (peau d'orange) in the vicinity of the areola. Which of the following best explains the inversion of her nipple? A. Retention of the fetal and infantile state of the nipple B. Intraductal cancerous tumor C. Retraction of the suspensory ligaments of the breast by cancer D. Obstruction of the cutaneous lymphatics, with edema of the skin E. Inflammation of the epithelial lining of the nipple and underlying hypodermis

C. Retraction of the suspensory ligaments of the breast by cancer The patient's symptoms are all indicative of inflammatory breast cancer. Common symptoms include inversion of the nipple and dimpling of the overlying skin, changes that are due to the retraction of the suspensory ligaments (of Cooper).

A 72-year-old male is admitted to the hospital with severe chest pain. ECG examination provides evidence of severe myocardial infarction of the lower part of the muscular interventricular septum. The function of which of the following valves will be most severely affected? A. Pulmonary B. Aortic C. Tricuspid D. Mitral E. Eustachian

C. Tricuspid The interventricular septum is intimately involved with the tricuspid valve on the right side, via the muscular connections of the septomarginal trabeculum (moderator band) to the anterior papillary muscle. Therefore, if the electrical system of the heart is disrupted, as with a myocardial infarction in the upper portion of the muscular septum, the innervation of the interventricular septum will be compromised and the tricuspid valve will be directly affected.

A 43-year-old woman is diagnosed with mitral valve stenosis. During physical examination the first heart sound is abnormally loud. Which of the following heart valves are responsible for the production of the first heart sound? A. Aortic and mitral B. Aortic and tricuspid C. Tricuspid and mitral D. Mitral and pulmonary E. Tricuspid and pulmonary

C. Tricuspid and mitral The first heart sound is caused by the closure of the tricuspid and mitral valves.

A 15-year-old male is admitted to the hospital with cough and severe dyspnea. Physical examination reveals expiratory wheezes, and a diagnosis is made of acute asthma. The expiratory wheezes are characteristic signs of bronchospasm of the smooth muscle of the bronchial airways. Which of the following nerves could be blocked to result in relaxation of the smooth muscle? A. Phrenic B. Intercostal C. Vagus D. T1 to T4 sympathetic fibers E. Recurrent laryngeal nerve

C. Vagus Bronchial constriction is induced by the parasympathetic innervation of the airways. This is supplied by the vagus nerves, which could be blocked to result in relaxation of the airways.

A 35-year-old female is admitted to the emergency department because of cardiac arrhythmia. ECG examination reveals that the patient suffers from atrial fibrillation. Where is the mass of specialized conduct- ing tissue that initiates the cardiac cycle located? A. At the junction of the coronary sinus and the right atrium B. At the junction of the inferior vena cava and the right atrium C. At the junction of the superior vena cava and the right atrium D. Between the left and right atria E. In the interventricular septum

C. at the junction of the superior vena cava and the right atrium The SA node, the primary pacemaker of the heart, is a mass of specialized cardiac cells within the myocardium at the upper end of the crista terminalis, near the opening of the superior vena cava into the right atrium.

A 3-month-old infant is diagnosed with a membranous ventricular septal defect. A cardiac operation is performed, and the septal defect is patched inferior to the noncoronary cusp of the aorta. Two days postoperatively the infant develops severe arrhythmias affecting both ventricles. Which part of the conduction tissue was most likely injured during the procedure? A. Right bundle branch B. Left bundle branch C. Bundle of His D. Posterior internodal pathway E. Atrioventricular node

C. bundle of his The bundle of His is a collection of specialized cardiac muscle cells that carry electrical activity to the right and left bundle branches. Because both ventricles are affected, this is the logical site of injury, for this bundle leads to the bundle branches supplying both ventricles.

A 49-year-old woman is admitted to the hospital complaining of severe, crushing, retrosternal pain during the preceding hour. An ECG reveals that she is suffering from acute myocardial infarction in the posterior aspect of her left ventricle and posteromedial papillary muscle. A coronary angiogram is performed and the patient is found to have left coronary dominant circulation. Which of the following arteries is the most likely to be occluded? A. Artery of the conus B. Right coronary artery C. Circumflex D. Right acute marginal E. Left diagonal

C. circumflex A "left coronary dominant" circulation means, most simply, that the left coronary artery (LCA) provides the posterior interventricular artery as a terminal branch of the coronary circumflex. The posterior aspect of the heart is composed primarily of the left ventricle and is supplied by the posterior interventricular branch.

A 72-year-old male is admitted to the hospital with complaints of severe chest pain radiating to his left arm. ECG examination provides evidence of significant myocardial infarction of the posterior wall of the left ventricle. Which of the following nerves is responsible for the radiation of pain to the arm during myocardial infarction? A. Phrenic B. Vagus C. Intercostobrachial D. Greater splanchnic E. Suprascapular

C. intercostobrachial The intercostobrachial nerve is the lateral cutaneous branch of the second intercostal nerve. It serves a sensory function both in the thoracic wall and medial aspect of the arm

A 32-year-old female is admitted to the emergency department with dyspnea, dysphagia, hoarseness, and severe anxiety. Her medical history reveals that she has lived on a liquid diet for some months and has lost more than 30 lb. Over the past several weeks, she has had bloody sputum during attacks of coughing. Fluoroscopy and a barium swallow reveal a 4-cm mass associated with a bronchus and associated compression of the esophagus. Which of the following nerves is most likely to be affected? A. Right recurrent laryngeal nerve B. Left vagus nerve, posterior to the hilum of the lung C. Left recurrent laryngeal nerve D. Greater thoracic splanchnic nerve E. Phrenic nerve

C. left recurrent laryngeal nerve The left recurrent laryngeal nerve passes superiorly in the tracheoesophageal groove after looping around the aorta. The compression of this nerve and compression of the esophagus against the trachea would result in the presenting symptoms.

A 42-year-old woman is admitted to the hospital with an inability to speak. The patient's personal history reveals that she has experienced hoarseness for the past month. A chest radiograph reveals a mass at the aortopulmonary window. Which of the following nerves is most likely compressed? A. Vagus B. Phrenic C. Left recurrent laryngeal D. Right recurrent laryngeal E. Greater thoracic splanchnic

C. left recurrent laryngeal nerve There is close proximity between the aortopulmonary window and the left recurrent laryngeal nerve. A mass within or adjacent to this window is thus likely to compress the left recurrent laryngeal nerve, resulting in the hoarseness for the patient.

A 62-year-old male patient expresses concern that his voice has changed over the preceding months. Im- aging reveals a growth located within the aortic arch, adjacent to the left pulmonary artery. Which neural structure is most likely being compressed to cause the changes in the patient's voice? A. Left phrenic nerve B. Esophageal plexus C. Left recurrent laryngeal nerve D. Left vagus nerve E. Left sympathetic trunk

C. left recurrent laryngeal nerve wraps around the aortic arch

A 17-year-old girl is admitted to the hospital with severe dyspnea. Physical examination reveals that the patient is suffering from an asthma attack, with associated bronchospasm. Which of the following nerves is responsible for the innervation of the bronchial smooth muscle cells? A. Greater thoracic splanchnic B. Phrenic C. Vagus D. Intercostal E. Lesser thoracic splanchnic

C. vagus The vagus nerve is the only nerve responsible for parasympathetic innervation of the lungs.

A 35-year-old woman is admitted to a surgical ward with a palpable mass in her right breast and swollen lymph nodes in the axilla. Radiographic studies and biopsy reveal carcinoma of the breast. Which group of axillary lymph nodes is the first to receive lymph drainage from the secretory tissue of the breast and therefore most likely to contain metastasized tumor cells? A. Lateral B. Central C. Apical D. Anterior (pectoral) E. Posterior (subscapular)

D. Anterior (Pectoral) Lymphatic drainage of the breast is typically to the axillary nodes, more specifically to the anterior (pectoral) nodes.

A 62-year-old male was admitted to the hospital with intense left chest pain. ECG and echocardiography reveal myocardial infarction and pulmonary valve regurgitation. Emergency coronary angiography is performed and provides evidence that the artery supplying the upper portion of the anterior right ventricular free wall is occluded. Which of the following arteries is most likely to be occluded? A. Circumflex B. Anterior interventricular artery C. Posterior interventricular artery D. Artery of the conus E. Acute marginal branch of the right coronary artery

D. Artery of the conus The artery of the conus is given off from the right coronary artery and winds around the conus arteriosus. The conus region is the superior part of the right ventricle that tapers into a cone (infundibulum) where the pulmonary valve leads into the pulmonary trunk. This conus artery supplies the upper portion of the anterior right ventricle and usually has a small anastomotic connection with the anterior interventricular (left anterior descending) branch of the left coronary artery.

A 58-year-old female with cardiac arrhythmia has undergone a procedure to implant a pacemaker. The electrical conducting leads for the pacemaker must be passed into the heart from the pacemaker. Which of the following is the correct order of structures for passage of the leads into the right ventricle? A. Brachiocephalic vein, superior vena cava, mitral valve, right ventricle B. Superior vena cava, right atrium, mitral valve, right ventricle C. Superior vena cava, right atrium, tricuspid valve, right ventricle D. Brachiocephalic vein, superior vena cava, right atrium, tricuspid valve, right ventricle E. Brachiocephalic vein, superior vena cava, right atrium, mitral valve, right ventricle

D. Brachiocephalic vein, superior vena cava, right atrium, tricuspid valve, right ventricle The correct path that leads to the right ventricle for the lead of the pacemaker is the brachiocephalic vein (could be right or left; pacemakers are more commonly placed on the left so would be left brachiocephalic vein), superior vena cava, right atrium, tricuspid valve, and right ventricle.

A 55-year-old male is admitted to the emergency department with a diagnosis of possible myocardial infarction. Which nerves carry pain fibers from the heart to the CNS? A. Vagus B. Greater thoracic splanchnic C. Least thoracic splanchnic D. Cardiopulmonary (thoracic visceral) E. T5 to T9 ventral rami

D. Cardiopulmonary nerves The cardiopulmonary splanchnic (or thoracic visceral) nerves are responsible for carrying the cardiac sympathetic efferent fibers from the sympathetic ganglia to the thoracic viscera and afferent fibers for pain from these organs.

A 42-year-old female is admitted to the hospital with dyspnea. Imaging reveals severe mitral valve regurgitation. Which of the following structures prevents regurgitation of the mitral valve cusps into the left atrium during systole? A. Crista terminalis B. Crista supraventricularis C. Pectinate muscles D. Chordae tendineae E. Trabeculae carneae

D. Chordae tendineae The chordae tendineae are fibrous cords that connect papillary muscles to valve leaflets. The restraint provided by these cords on the valve leaflets prevents the prolapse of the mitral valve cusps into the left atrium.

A 25-year-old man is admitted to the emergency department with a bullet wound in the neck just above the middle of the right clavicle and first rib. Radiographic examination reveals collapse of the right lung and a tension pneumothorax. Injury to which of the following respiratory structures resulted in the pneumothorax? A. Costal pleura B. Cupula C. Right mainstem bronchus D. Right upper lobe bronchus E. Mediastinal parietal pleura

D. Cupula Another name for the cervical pleura is the cupula. This forms the dome of the plura, projecting into the neck above the first rib and corresponding to the area of injury.

A 3-year-old child is admitted to the emergency department with a particularly severe attack of asthma. Which of the following is the most important factor in increasing the intrathoracic capacity in inspiration? A. "Pump handle movement" of the ribs— thereby increasing anterior-posterior dimensions of the thorax B. "Bucket handle movement" of the ribs— increasing the transverse diameter of the thorax C. Straightening of the forward curvature of the thoracic spine, thereby increasing the vertical dimensions of the thoracic cavity D. Descent of the diaphragm, with protrusion of the abdominal wall, thereby increasing vertical dimensions of the thoracic cavity E. Orientation and flexibility of the ribs in the baby, thus allowing expansion in all directions

D. Descent of the diaphragm, with protrusion of the abdominal wall, thereby increasing vertical dimensions of the thoracic cavity Contraction of the diaphragm (descent) pulls the dome inferiorly, increasing the vertical dimension of the thorax. This is the most important factor in inspiration for increasing the internal pulmonary volume and concomitantly decreasing intrathoracic pressure.

Ten days after a surgical procedure to correct her cardiac malformation, a 3-month-old infant died unexpectedly in her sleep. After an autopsy, the pathologist reported as follows: "A significant portion of the conduction tissue was found to be necrotic. The area of the necrotic tissue was located inferior to the central fibrous body, membranous septum, and septal leaflet of the tricuspid valve. Further examination revealed infarction of the surrounding tissue. The rest of the heart was unremarkable." Which of the following arteries was most likely occluded? A. Artery of the conus B. SA node artery C. AV node artery D. First septal perforator of the anterior interventricular artery E. All of the above

D. First septal perforator of the anterior interventricular artery The first septal perforating branch of the anterior interventricular artery (left anterior descending: LAD) is the first branch of the LAD that supplies the conducting tissue of the heart; it passes directly to the point of bifurcation of the common atrioventricular bundle of His.

A 60-year-old man is admitted to the hospital with severe abdominal pain. A CT scan reveals a dissecting aneurysm of the thoracic aorta. While in the hospital the patient's aneurysm ruptures and he is transferred urgently to the operating theater. Postoperatively, the patient suffers from paraplegia. Which of the following arteries was most likely injured during the operation to result in the paralysis? A. Right coronary artery B. Left common carotid C. Right subclavian D. Great radicular (of Adamkiewicz) E. Esophageal

D. Great radicular The artery of Adamkiewicz (great radicular artery) is an important artery that provides oxygenated blood to the lower portion of the spinal cord, specifically the anterior cord where lower motor neurons are located, inferior to the vertebral level of origin of the artery, and provides collateral anastomoses with the anterior spinal artery. Care should be taken during surgery to prevent damage to this artery as this can lead to paraplegia and alteration of functions of pelvic organs.

A 42-year-old male is diagnosed with liver and pancreatic disease as a result of alcoholism. During physical examination it is noted that he has abnormal enlargement of his mammary glands, as a secondary result of his disease process. Which of the following clinical conditions will most likely describe this case? A. Polythelia B. Supernumerary breast C. Polymastia D. Gynecomastia E. Amastia

D. Gynecomastia Gynecomastia is the abnormal growth of mammary glands in males.

A 57-year-old male is admitted to the emergency department after he was struck by a truck while crossing a busy street. Radiographic examination reveals flail chest. During physical examination the patient complains of severe pain during inspiration and expiration. Which of the following nerves is most likely responsible for the sensation of pain during respiration? A. Phrenic B. Vagus C. Cardiopulmonary D. Intercostal E. Thoracic splanchnic

D. Intercostal Flail chest is characterized by paradoxical breathing movements caused by multiple rib fractures. The sensory innervation provided to intercostal spaces and to the underlying parietal pleura is supplied via the corresponding intercostal nerves.

A 54-year-old male is admitted to the hospital with severe chest pain. ECG examination reveals a myocardial infarction. If the posterior interventricular branch in the patient arises from the right coronary artery, which part of the myocardium will most likely have its blood supply reduced if the circumflex branch of the left coronary artery becomes occluded from an atherosclerotic plaque? A. Anterior part of the interventricular septum B. Diaphragmatic surface of the right ventricle C. Infundibulum D. Lateral wall of the left ventricle E. Posterior part of the interventricular septum

D. Lateral wall of the left ventricle The left coronary artery bifurcates into the anterior interventricular artery (left anterior descending: LAD) and the coronary circumflex branch. The circumflex branch gives off the left marginal branch, which supplies the lateral wall (obtuse margin) of the left ventricle.

A 10-year-old boy is admitted to the hospital with retrosternal discomfort. A CT scan reveals a midline tumor of the thymus gland. Which of the following veins would most likely be compressed by the tumor? A. Right internal jugular B. Left internal jugular C. Right brachiocephalic D. Left brachiocephalic E. Right subclavian

D. Left brachiocephalic The thymus lies in the superior mediastinum and extends upward into the neck, especially in the young. A midline tumor of this gland can compress the left brachiocephalic vein.

A 22-year-old man is diagnosed with signs of reduced aortic flow. Upon examination it is noted that brachial artery pressure is markedly increased, femoral pressure is decreased, and the femoral pulses are delayed. The patient shows no external signs of inflammation. Which of the following conditions will most likely be observed in a radiographic examination? A. Flail chest B. Pneumothorax C. Hydrothorax D. Notching of the ribs E. Mediastinal shift

D. Notching of the ribs The diagnosis for these symptoms is coarcta- tion of the aorta. This condition occurs when the aorta is abnormally constricted. One of the cardinal signs is a characteristic rib notching. "Notching" of the ribs is due to the reversal of direction of blood flow through the anterior intercostal branches of the internal thoracic artery, as these usually small arteries carry collateral arterial blood flow to the lower thoracic portion of the aorta inferior to the coarctation. Enlargement and vibration of the intercostal arteries against the rib results in erosion ("notching") of the subcostal grooves, which is visible on radiography.

A 69-year-old male is admitted to the hospital with intense left chest pain. ECG reveals hypokinetic ventricular septal muscle, myocardial infarction in the anterior two thirds of the interventricular septum, and left anterior ventricular wall. The patient's ECG also exhibited left bundle branch block. Which of the following arteries is most likely occluded? A. Circumflex B. Proximal right coronary C. Proximal left coronary D. Proximal left anterior interventricular artery E. Posterior interventricular artery

D. Proximal left anterior interventricular artery The tissues affected in this case, the interventricular septum and anterior ventricular wall, are mostly supplied by the proximal portion of the left anterior interventricular artery.

During cardiac surgery of a 45-year-old male the cardiac surgeon can place her fingers in the transverse pericardial sinus, if necessary. This allows the surgeon to easily place a vascular clamp upon which of the fol- lowing vessels? A. Right and left pulmonary veins B. Superior and inferior vena cava C. Right and left coronary arteries D. Pulmonary trunk and ascending aorta E. Pulmonary trunk and superior vena cava

D. Pulmonary Trunk and ascending aorta A finger passing through the transverse pericardial sinus passes directly behind the great arteries exiting the heart, allowing the surgeon to rather easily place a vascular clamp upon the pulmonary trunk and ascending aorta.

A 45-year-old female is admitted to the hospital with swelling (edema) of the lower limbs. Ultrasound examination reveals an incompetent tricuspid valve. Into which area will regurgitation of blood occur in this patient? A. Pulmonary trunk B. Left atrium C. Ascending aorta D. Right atrium E. Left ventricle

D. Right atrium The tricuspid valve is the atrioventricular valve located between the right atrium and right ventricle. An incompetent valve would allow blood to regurgitate into the right atrium during systole and subsequently raise pressure in the venous system, increasing capillary pressure and causing edema.

A 42-year-old female is admitted urgently to the emergency department after suffering a penetrating wound to her chest from an ice pick during a violent domestic dispute. Physical and ultrasound examinations reveal that the patient has cardiac tamponade. Which of the following will most likely be found during physical examination? A. There will be a visible or palpable decrease in the dimensions of the external jugular and internal jugular vein. B. There will be gradual enlargement of the ventricles in diastole. C. The difference between systolic and diastolic arterial pressures will increase significantly. D. There will be diminished heart sounds. E. The pulses in the internal carotid arteries will become increasingly distinct, as detected be- hind the angles of the mandible.

D. There will be diminished heart sounds Cardiac tamponade is characterized by hypo- tension, tachycardia, muffled heart sounds, and jugular vein distention. Bleeding into the pericardial cavity would muffle the heart sounds because of the increased distance between the chest wall and the heart, leading to "distant" heart sounds. When the effusion is particularly severe, the heart may take on a "water bottle" appearance on an anterior-posterior radiograph.

A 59-year-old man is admitted to the hospital with severe chest pain. During examination a slight rhythmic pulsation on the chest wall at the left fifth intercostal space is noted in the midclavicular line. What part of the heart is responsible for this pulsation? A. Right atrium B. Left atrium C. Aortic arch D. Apex of the heart E. Mitral valve

D. apex of the heart The apex of the heart is located in the left fi fth intercostal space, about 3 1 ⁄ 2inches to the left of the sternum. When this area of the heart is palpated, any pulsations would be generated by throbbing of the apex of the heart against the thoracic wall. This is also the location for performing auscultation (listening) of the mitral valve, not associated with palpation.

A 43-year-old male hunter is admitted to the emergency department after falling over a barbed wire fence, as a result of which he suffered several deep lacerations along the left midaxillary line. When the patient is examined in the outpatient clinic several days later, numbness and anhydrosis are observed anterior to the area of the cuts. Which structures were most likely damaged to result in these signs? A. Dorsal roots B. Ventral roots C. Cutaneous branches of dorsal rami D. Cutaneous branches of ventral rami E. Rami communicans

D. cutaneous branches of ventral rami Ventral rami contain both sensory and motor fibers and also sympathetics to the body wall, supplying all areas of the body wall except for tissues of the back. In this case both sensory fibers (numbness) and sympathetics (anhydrosis) are disrupted at the midaxillary line; therefore, cutaneous ventral rami is the only correct choice.

A 23-year-old man is admitted to the emergency department after an automobile collision. Physical examination reveals tachycardia. What is the location of the preganglionic neural cell bodies involved in increasing the heart rate? A. Deep cardiac plexus B. Dorsal motor nucleus of vagus C. Lateral horn T5 to T9 D. Lateral horn T1 to T4 E. Superior, middle, and inferior cervical ganglia

D. lateral horn of T1 to T4 The lateral horns, or intermediolateral cell columns, contain the cell bodies of preganglionic neurons of the sympathetic system. Spinal cord segments T1 to T4 are often associated with the upper limbs and thoracic organs

A 62-year-old patient is admitted to the hospital with a complaint of suddenly occurring, tearing pain radiating to his back. A CT examination reveals that the patient has an aortic aneurysm. An urgent placement of an endovascular stent-graft is ordered. Which of the following nerves are most likely responsible for the tearing sensation radiating to his back? A. Somatic afferent B. Thoracic visceral afferent C. Sympathetic postganglionics D. Sympathetic preganglionics E. Parasympathetic afferent

D. sympathetic preganglionics General visceral afferents are nerve fibers that carry sensation from organs, in this case pain from the abdominal aorta. These fibers get mixed with general somatic afferents in the dorsal roots. This is the phenomenon of "referred pain." The dorsal root ganglia (or their counterparts associated with sensory cranial nerves) contain the cell bodies associated with all sensory fibers from the body, including somatic and visceral sensation.

A 3-month-old male infant died unexpectedly in his sleep. The pathologist examined the histologic slides of tissue samples taken from the heart of the infant and observed that a portion of the conduction tissue that penetrates the right fibrous trigone had become necrotic. As a result, a fatal arrhythmia probably developed, leading to the death of the infant. Which of the following parts of the conduction tissue was most likely interrupted? A. Right bundle branch B. The bundle of Bachmann C. The left bundle branch D. The atrioventricular bundle of His E. The posterior internodal pathway

D. the atrioventricular bundle of his The atrioventricular bundle of His is a strand of specialized cardiac muscle fibers that arises from the atrioventricular node and passes through the right fibrous trigone. The right fibrous trigone (central fibrous body) is a dense area of connective tissue that interconnects the mitral, tricuspid, and aortic valve rings. After reaching the upper portion of the muscular interventricular septum, the bundle of His splits into right and left bundle branches.

A 55-year-old male is admitted to the emergency department with severe chest pain. Coronary angiography reveals that the patient's right coronary artery is free of pathology. The left coronary artery is found to be 70% to 80% occluded at three points proximal to its bifurcation into the circumflex and left anterior descending arteries. Having a left dominant coronary circulation, and without surgery, what is the most likely explanation for a poor prognosis for recovery of this patient to a normally active life? A. All the branches of the coronary artery are end arteries, precluding the chance that anastomotic connections will occur. B. It is probable that the anterior and posterior papillary muscles of the tricuspid valve have been damaged. C. The blood supply of the SA node is inadequate. D. The development of effective collateral circulation between anterior and posterior interventricular arteries will not be possible. E. The blood supply of the AV node will be inadequate.

D. the development of effective collateral circulation between anterior and posterior interventricular arteries will not be possible Because the patient is left coronary artery dominant, if there is 70% to 80% occlusion of the left coronary, there will be deficiencies in fl ow both in the anterior descending and circumflex coronary arteries.

A 58-year-old patient presents himself to the emergency department with severe angina. Upon cardiac catheterization, it is found that he has a significant occlusion in his right coronary artery, just distal to the right sinus of the aortic valve. His collateral cardiac circulation is minimal. Assuming the patient is right coronary dominant, which of the following arteries would be most likely to still have normal blood flow? A. Right (acute) marginal artery B. Atrioventricular nodal artery C. Posterior interventricular artery D. Sinoatrial nodal artery E. Anterior interventricular artery

E. Anterior interventricular artery The anterior interventricular artery arises from the left coronary artery. If there is occlusion in the right coronary artery, the anterior interventricular artery will still have normal blood flow.

During surgical repair of a congenital cardiac anomaly in a 15-year-old boy with a right dominant coronary arterial system, the surgeon accidentally injured a vessel that usually supplies part of the conduction system. This results in intermittent periods of atrioventricular block and severe arrhythmia. The injured artery was most likely a direct branch of which of the following arteries? A. Distal anterior interventricular artery B. Circumflex artery C. Left coronary artery D. Marginal artery E. Right coronary artery

E. Right coronary artery "Right coronary dominant circulation" refers simply to the fact that the right coronary artery provides origin for the posterior interventricular (poste- rior descending) coronary artery. In such cases, it provides supply for the sinoatrial and atrioventricular nodes.

A 21-year-old female gymnast is admitted to the hospital with severe dyspnea after a fall from the un- even parallel bars. Radiographic examination reveals that her right lung is collapsed and the left lung is com- pressed by the great volume of air in her right pleural cavity. During physical examination she has no signs of external injuries. Which of the following conditions will most likely describe this case? A. Flail chest with paradoxical respiration B. Emphysema C. Hemothorax D. Chylothorax E. Tension pneumothorax

E. Tension pneumothorax A tension pneumothorax is caused by injury to the lung, leading to air in the pleural cavity. The site of the wound acts as a one-way valve, allowing air to enter the pleural cavity but not to leave the cavity. The lack of negative pressure in the pleural cavity causes the lung to collapse.

A 35-year-old female is admitted to the hospital with dyspnea. During physical examination her S 1 heart sound is very loud. Which of the following valves is/are responsible for production of the S 1 heart sound? A. Mitral valve B. Pulmonary and aortic C. Aortic and mitral D. Tricuspid E. Tricuspid and mitral

E. Tricuspid and Mitral The closure of the mitral/bicuspid and tricuspid valves produces the first S 1 (lub) heart sound.

A 32-year-old female is admitted to the hospital in a comatose state. Physical examination reveals that the patient suffers from anorexia nervosa. A nasogastric tube is ordered to be inserted. What is the last site at which resistance would be expected as the tube passes from the nose to the stomach? A. Pharyngoesophageal junction B. Level of the superior thoracic aperture C. Posterior to the aortic arch D. Posterior to the left main bronchus E. Esophageal hiatus of the diaphragm

E. esophageal hiatus of the diaphragm The esophageal hiatus in the diaphragm is one of four openings associated with the diaphragm. It is located at the level of T10 and allows the esophagus to pass through the thoracic cavity into the abdominal cavity. It is the most inferior of four esophageal constrictions.

After a mastectomy, a musculocutaneous flap is used to restore the thoracic contour in a 34-year-old female patient. The ipsilateral (same side) rectus abdominis muscle was detached carefully from the surrounding structures and transposed to the thoracic wall. Which of the following landmarks is most often used to locate the inferior end of the posterior, tendinous layer of the rectus sheath? A. Intercristal line B. Linea alba C. Arcuate line D. Pectineal line E. Semilunar line

C. Arcuate line The arcuate line is a horizontal line that demarcates the lower limit of the posterior aponeurotic portion of the rectus sheath. It is also where the inferior epigastric vessels perforate the sheath to enter the rectus abdominis.

A 74-year-old woman is admitted to the hospital with complaints of abdominal pain. Radiographic examination reveals diverticulosis and diverticulitis of the lower portion of the descending colon, with diffuse ulcerations. It is determined that the involved area of the bowel should be removed. If the patient's anatomy follows the most typical patterns, which vessels and nerves will be cut during the operation? A. Branches of the vagus nerve and middle colic artery B. Superior mesenteric plexus and superior rectal artery C. Branches of pelvic splanchnic nerves and left colic artery D. Branches of vagus nerve and ileocolic artery E. Left thoracic splanchnic nerve and inferior mesenteric artery

C. Branches of pelvic splanchnic nerves and left colic artery Pelvic splanchnic nerves and the left colic artery supply the descending colon.

A 45-year-old male entered the emergency depart- ment with a complaint of severe abdominal pain. During physical examination it is observed that his cremasteric reflex is absent. Which of the following nerves is responsible for the efferent limb of the cremasteric reflex? A. Ilioinguinal B. Iliohypogastric C. Genitofemoral D. Pudendal E. Ventral ramus of T12

C. Genitofemoral The genitofemoral nerve originates from the ventral rami of L1 and L2. The femoral part supplies skin to the femoral triangle area, whereas the "genito" part in males travels with the spermatic cord and supplies the cremaster muscle and scrotal skin.

A hard mass (a fecolith) in the ostium of the 27-year-old patient's appendix had led to a local infection (appendicitis) with a slightly elevated temperature and a moderate increase in WBC count. The initial pain from the infection was dull and difficult to localize, but the patient placed his hand in the periumbilical area to indicate the general area of discomfort. The region of the umbilicus receives its sensory supply, classically, from which of the following spinal nerves? A. T7 B. T8 C. T10 D. T12 E. L1

C. T10 The dermatome of spinal nerve level T10 crosses the level of the umbilicus;

A 53-year-old quality control engineer is admitted to the hospital with severe abdominal pain. The patient's history and physical examination indicate chronic colonic diverticulitis, with tachycardia and hypotension at the present time. An ultrasonographic examination reveals massive bleeding from the descending colon. Which of the following arteries is most likely to be the source of the hemorrhage? A. A branch of the inferior mesenteric B. Middle colic C. Superior rectal D. Inferior rectal E. Left gastroepiploic

A. A branch of the inferior mesenteric The most likely source of this hemorrhage is from the left colic artery. Colonic diverticular disease is the development of blind end sacs from the wall of the colon. Of the selected choices the left colic artery is the only artery supplying a portion of the descending colon.

A 24-year-old woman has a dull aching pain in the umbilical region, and flexion of the hip against re- sistance (psoas test) causes a sharp pain in the right lower abdominal quadrant. Which of the following structures is most likely inflamed to cause the pain? A. Appendix B. Bladder C. Gallbladder D. Pancreas E. Uterus

A. Appendix The appendix is the most likely structure that is inflamed. It lies in the right lower quadrant, and of the choices provided, it is most closely associated with the umbilical region by way of referral of pain. The patient also exhibited a positive psoas sign when flexion of the hip against resistance was attempted. This is because the iliopsoas muscle group lies directly beneath the appendix, and upon flexion of this muscle group, contact and direct irritation to the appendix can occur.

A 63-year-old man with a history of alcoholism is brought to the emergency department with hemateme- sis (vomiting blood). Findings on endoscopic examina- tion suggest bleeding from esophageal varices. The varices are most likely a result of the anastomoses be- tween the left gastric vein and which other vessel or vessels? A. Azygos system of veins B. Inferior vena cava C. Left umbilical vein D. Superior mesenteric vein E. Subcostal veins

A. Azygos system of veins Esophageal varices are dilated veins in the submucosa of the lower esophagus. They often result from portal hypertension due to liver cirrhosis. The left gastric vein and the esophageal veins of the azygos system form an important portal-caval anastomosis when pressure in the portal vein, and in turn the left gastric vein, is increased.

A 43-year-old woman is admitted to the emergency department with esophageal pain and hematemesis after swallowing a fish bone. An endoscopic examination reveals perforation of the intraabdominal portion of the esophageal wall. Which of the following arteries is most likely injured? A. Branches of left gastric B. Bronchial C. Thoracic intercostal D. Branches of right gastric E. Right inferior phrenic

A. Branches of left gastric The lowest part of the esophagus, below the diaphragm, is supplied by the left gastric artery. Perforation to this area could easily injure this artery.

A 35-year-old obese man is admitted to the hospital with jaundice and complaints of abdominal pain. Physical examination reveals an epigastric pain that migrates toward the patient's right side and posterior toward the scapula. Radiographic examination reveals multiple gallstones, consistent with the patient's jaundice and typical pains of cholecystitis. Which of the following structures is most likely obstructed by the gallstones? A. Common bile duct B. Cystic duct C. Left hepatic duct D. Pancreatic duct E. Right hepatic duct

A. Common bile duct The common bile duct is occluded. The pattern of pain of cholecystitis (and other signs), combined with jaundice, indicates blockage of release of bile into the duodenum.

A 62-year-old man is admitted to the hospital with dull, diffuse abdominal pain. A CT scan reveals a tumor at the head of the pancreas. The abdominal pain is mediated by afferent fibers that travel initially with which of the following nerves? A. Greater thoracic splanchnic B. Intercostal C. Phrenic D. Vagus E. Subcostal

A. Greater thoracic splanchnic The afferent fibers mediating the pain from the head of the pancreas run initially with the greater thoracic splanchnic nerves. The greater splanchnic nerves arise from sympathetic ganglia at the levels of T5 to T9 and innervate structures of the foregut and thus the head of the pancreas. Running within these nerves are visceral afferent fibers that relay pain from foregut structures to the dorsal horn of the spinal cord.

A 62-year-old woman is admitted to the emergency department with abdominal pains of uncertain origin. A CT scan reveals an aortic aneurysm affecting the origin of the superior mesenteric artery, resulting in ischemia to an abdominal organ. Which of the following organs is most likely affected? A. Ileum B. Transverse colon C. Spleen D. Stomach E. Duodenum

A. Ileum The ileum can become ischemic when arterial supply from the superior mesenteric artery is compromised. The superior mesenteric artery arises from the aorta posterior to the neck of the pancreas. It descends across the third part of the duodenum and enters the root of the mesentery behind the transverse colon. This artery gives origin to the following branches: inferior pancreaticoduodenal artery, middle colic artery, ileocolic artery, right colic artery, and intestinal arteries. The ileocolic artery descends behind the peritoneum toward the right and ends by dividing into the ascending colic artery, anterior and posterior cecal arteries, the appendicular artery, and ileal branches. The ileum is supplied by the ileal branches, which do not have any anastomoses with another major source vessel.

A 45-year-old male is admitted to the hospital with a massive hernia that passes through the inguinal triangle (of Hesselbach). Which of the following structures is used to distinguish a direct inguinal hernia from an indirect inguinal hernia? A. Inferior epigastric vessels B. Femoral canal C. Inguinal ligament D. Rectus abdominis muscle (lateral border) E. Pectineal ligament

A. Inferior epigastric vessels The key distinguishing feature of a direct inguinal hernia is that the direct hernia does not pass through the deep inguinal ring; it passes through the lower portion of the inguinal triangle (of Hesselbach). This triangle is bordered laterally by the inferior epigastric artery and vein; medially, it is bordered by the lateral edge of rectus abdominis; inferiorly, it is bordered by the iliopubic tract and inguinal ligament.

Two months postoperatively the patient complained of abdominal pain. A CT scan examination demonstrated an internal hernia in which the hepatic flexure of the colon had herniated through the epiploic (omental) foramen (of Winslow). Gastrointestinal veins appeared to be markedly dilated, including the veins forming anastomoses between the portal and caval systems (veins of Retzius). Which of the following structures is most likely compressed? A. Portal vein B. Inferior vena cava C. Hepatic artery D. Common bile duct E. Cystic duct

A. Portal vein The portal vein is compressed in its passage through the hepatoduodenal ligament, the anterior border of the omental (epiploic) foramen (of Winslow). The veins of Retzius are located along the sides of the abdominal walls and communicate between tributaries of retroperitoneal parts of the gastrointestinal tract and veins of the body wall. In portal hypertension the portal blood cannot pass freely through the liver, and the portal-caval tributaries and their anastomoses become dilated. The inferior epigastric veins anastomose with the paraumbilical veins, which is the first branch off the hepatic portal vein. These would be the first affected in portal hypertension.

Radiographic examination of a 42-year-old female reveals penetration of the duodenal bulb by an ulcer, resulting in profuse intraabdominal bleeding. Which of the following arteries is the most likely source of the bleeding? A. Posterior superior pancreaticoduodenal B. Superior mesenteric C. Inferior mesenteric D. Inferior pancreaticoduodenal E. Right gastric

A. Posterior superior pancreaticoduodenal The posterior superior pancreaticoduodenal artery arises from the gastroduodenal artery and travels behind the first part of the duodenum, supplying the proximal portion, with branches to the head of the pancreas. Duodenal ulcers commonly arise within the first portion of the duodenum, thus making the posterior superior pancreaticoduodenal artery one of the more frequently injured vessels.

After a "tummy-tuck" (abdominoplasty) procedure is performed on a 45-year-old man, which of the follow- ing layers of the abdominal wall will hold the sutures? A. Scarpa's fascia (membranous layer) B. Camper's fascia (fatty layer) C. Transversalis fascia D. Extraperitoneal tissue E. External abdominal oblique fascia

A. Scarpas fascia Scarpa's fascia is the thick, membranous layer deep to the Camper's adipose fascia in the anterior abdominal wall (subcutaneous). Because of the relatively thick, tough nature of connective tissue that makes up Scarpa's fascia, this layer is typically the site to maintain sutures.

A 30-year-old female patient complains that she has been weak and easily fatigued over the past 6 months. She has a 3-month acute history of severe hypertension that has required treatment with antihypertensive medications. She has recently gained 4.5 kg (10 lb) and currently weighs 75 kg (165 lb). Her blood pressure is 170/100 mm Hg. Purple striae are seen over the abdomen on physical examination and she possesses a "buffalo hump." Fasting serum glucose concentration is 140 mg/dl. A CT scan of the abdomen shows a 6-cm mass immediately posterior to the inferior vena cava. Which of the following organs is the most likely origin of the mass? A. Suprarenal (adrenal) gland B. Appendix C. Gallbladder D. Ovary E. Uterus

A. Suprarenal gland The right adrenal gland is a retroperitoneal organ on the superomedial aspect of the right kidney, partially posterior to the inferior vena cava.

In performing a laparoscopic hernia repair on a 24-year-old female gymnast, the surgical resident observed the bright reflection provided by the tissues of the iliopubic tract. The iliopubic tract could be traced medially to the site of the femoral herniation. The ilio-pubic tract is characterized by which of the following statements? A. The iliopubic tract represents the aponeurotic origin of the transversus abdominis. B. The iliopubic tract forms the lateral border of the inguinal triangle (of Hesselbach). C. The iliopubic tract forms the lateral border of the femoral ring. D. The iliopubic tract is the part of the inguinal ligament that attaches to the pectineal ligament. E. The iliopubic tract is the lateral extension of the pectineal ligament.

A. The iliopubic tract represents the aponeurotic origin of the transversus abdominis. The iliopubic tract is a refl ective band of aponeurotic tissue of the origin of the transversus abdominis, when visualized with the laparoscope.

A 37-year-old woman is admitted to the hospital with signs of cholecystitis. A physical examination confirms the initial diagnosis and a cholecystectomy is planned. Which of the following landmarks will best describe the precise location of the gallbladder with respect to the body wall? A. The intersection of the right linea semilunaris with the ninth costal cartilage B. The intersection of the right linea semilunaris with the intertubercular plane C. To the right of the epigastric region D. Superiorly to the umbilical region E. Upper right quadrant

A. The intersection of the right linea semilunaris with the ninth costal cartilage The intersection of the right linea semilunaris with the ninth costal cartilage in the right upper quadrant is associated typically with the point of contact of the gallbladder fundus with the anterior abdominal wall.

A 38-year-old pregnant woman is admitted to the emergency department with severe vaginal bleeding. Ultrasound examination confirms the initial diagnosis of ectopic pregnancy. Which of the following is the most common site of an ectopic pregnancy? A. Uterine tubes B. Cervix C. Mesentery of the abdominal wall D. Lower part of uterine body overlapping the internal cervical os E. Fundus of the uterus

A. Uterine tube The most common site of ectopic pregnancy is in the uterine tubes.

A 47-year-old female patient's right breast exhibited peau d'orange characteristics. This condition is primarily a result of which of the following occurrences? A. Blockage of cutaneous lymphatic vessels B. Shortening of the suspensory ligaments by cancer in the axillary tail of the breast C. Contraction of the retinacula cutis of the are- ola and nipple D. Invasion of the pectoralis major by metastatic cancer E. Ipsilateral (same side) inversion of the nipple from cancer of the duct system of the breast

A. blockage of cutaneous lymphatic vessels Blockage of cutaneous lymphatic vessels results in edema of the skin surrounding the hair follicles, leading to an appearance like an orange peel (peau d'orange).

A 54-year-old male is admitted to the emergency department with severe upper abdominal pain. Gastroscopy reveals a tumor in the antrum of the stomach. A CT scan is ordered to evaluate lymphatic drainage of the stomach. Which of the following lymph nodes is most likely to be involved in a malignancy of the stomach? A. Celiac B. Superior mesenteric C. Inferior mesenteric D. Lumbar E. Hepatic

A. celiac The celiac lymph nodes receive lymph drainage directly from the stomach before they drain into the cisterna chyli.

A 49-year-old man presents with acute abdominal pain and jaundice. Radiographic studies reveal a tumor in the head of the pancreas. Which of the following structures is most likely being obstructed? A. Common bile duct B. Common hepatic duct C. Cystic duct D. Accessory pancreatic duct E. Proper hepatic artery

A. common bile duct The common bile duct is located at the head of the pancreas and receives contents from the cystic duct and hepatic duct. An obstruction at this site causes a backup of bile back through the common bile duct and hepatic duct, with resulting pain and jaundice.

An obese 45-year-old female patient with an elevated temperature comes to the physician's office com- plaining of nausea and intermittent, acute pain in the right upper quadrant of the abdomen during the past 2 days. She has a 24-hour history of jaundice. She has a history of gallstones. Which of the following structures has most likely been obstructed by a gallstone? A. Common bile duct B. Cystic duct C. Left hepatic duct D. Pancreatic duct E. Right hepatic duct

A. common bile duct The symptoms of yellow eyes and jaundice would be caused by reversal of flow of bile into the bloodstream. The common bile duct, if obstructed, allows no collateral pathway for drainage of bile from the liver or gallbladder.

Postoperative examination of a 68-year-old male who underwent mitral valve replacement demonstrates significant cardiac hypertrophy. Which of the following structures would be most likely to be compressed? A. Esophagus B. Pulmonary trunk C. Superior vena cava D. Trachea E. Inferior vena cava

A. esophagus Cardiac hypertrophy is a compensatory mechanism of the myocardium in response to increasing demands on the heart due to ischemia, incompetent valves, or hypertension. The increased size of the heart muscle would most likely compress the esophagus, and due to the incompetent mitral valve, a back flow of blood into the left atrium can cause a left atrial dilation. The left atrium lies just anteriorly to the esophagus in the mediastinum.

A 43-year-old woman visits the outpatient clinic with complaints of chronic dysphagia and gastroesophageal reflux. An endoscopic examination reveals achalasia of the cardia of the stomach. Which of the following is the most likely cause of this condition? A. Failure of relaxation of the lower esophageal sphincter B. Dyspepsia C. Gastritis D. Gastroparesis E. Peptic ulcer

A. failure of relaxation of the lower espohageal sphincter Failure of relaxation of the lower esophageal sphincter (also known as the cardiac sphincter) causes an accumulation of food in the esophagus. Achalasia is the failure of motility of food through the esophagus into the stomach. A constricted lower esophageal sphincter is the cause of these conditions.

A 53-year-old man is admitted to the hospital with rectal bleeding. Physical examination, including a rectal examination, reveals an abnormal mass of tissue protruding from an area superior to the external anal sphincter, superior to the pectinate line. Biopsy reveals the presence of an adenocarcinoma. Which of the following groups of lymph nodes would first receive lymph from the cancerous area? A. Internal iliac B. External iliac C. Middle rectal D. Superficial inguinal E. Deep inguinal

A. internal iliac The lymphatics of the inferior rectum above the pectinate line drain into the internal iliac nodes.

A 68-year-old woman had been suffering long-term effects of diverticulosis and inflammation of the transverse colon. To permit operating on a patient with severe diverticulosis of the transverse colon, it would be necessary to first ligate (tie off) or clamp the source of arterial supply. Which of the following arteries will most likely be ligated? A. Middle colic B. Right colic C. Superior mesenteric D. Ileocolic E. Left colic

A. middle colic The middle colic artery is the principal source of arterial supply to the transverse colon.

A 42-year-old male is admitted to the hospital with severe hematemesis. Radiographic studies reveal hepatomegaly and esophageal varices. During physical examination it is observed that the patient is icteric (jaundiced) and dilated veins ("caput medusae") are seen on his anterior abdominal wall. Which of the fol- lowing venous structures is most likely obstructed for the development of caput medusae? A. Portal vein B. Inferior vena cava C. Superior vena cava D. Lateral thoracic vein E. Superficial epigastric vein

A. portal vein Caput medusae is caused by severely elevated portal pressure, with venous reflux from the liver to the periumbilical veins, by way of the usually collapsed veins in the ligamentum teres. Caput medusae is identified by the appearance of engorged veins radiating toward the lower limbs. The portal vein is the central connection of these anastomoses.

A 30-year-old woman complains of weakness and fatigability over the past 6 months. She has a 3-month acute history of severe hypertension that has required treatment with antihypertensive medications. Radiographic examination reveals a tumor of her right suprarenal gland. The patient is diagnosed with a pheochromocytoma (tumor of the adrenal medulla) and is scheduled for a laparoscopic adrenalectomy. Which of the following nerve fibers will need to be cut when the adrenal gland and tumor are removed? A. Preganglionic sympathetic fi bers B. Postganglionic sympathetic fi bers C. Somatic motor fi bers D. Postganglionic parasympathetic fi bers E. Preganglionic parasympathetic fi bers

A. preganglionic sympathetic fibers The preganglionic sympathetic fibers running to the adrenal gland would be cut during adrenalectomy for they synapse on catecholamine-secreting cells within the adrenal medulla. Unlike the normal route of sympathetic innervation, which is to first synapse in a sympathetic ganglion and then send postganglionic fibers to the target tissue, the chromaffin cells of the adrenal gland are innervated directly by preganglionic sympathetic fibers. This is because the chromaffin cells are embryologically postganglionic neurons that migrate to the medulla and undergo differentiation.

In a routine visit to the outpatient clinic for his annual checkup, a 42-year-old male is informed that radiographic examination has given strong evidence that he has a malignancy of his scrotum. Which of the following nodes are the first lymph nodes that drain the affected area? A. Superficial inguinal B. Internal iliac C. Lumbar D. Presacral E. Axillary

A. superficial inguinal The lymph drainage of the scrotum is into the superficial inguinal nodes.

A 65-year-old man is admitted to the emergency department with complaints of nonspecific abdominal pain. Physical and radiographic examinations reveal mild intestinal ischemia due to atherosclerotic occlusion of the midproximal part of the superior mesenteric artery, but collateral blood supply has delayed the onset of necrosis. What vessels provide collateral channels between the celiac trunk and the superior mesenteric artery? A. Superior and inferior pancreaticoduodenal B. Left gastric and hepatic C. Cystic and gastroduodenal D. Right and left colic E. Right and left gastroomental

A. superior and inferior pancreaticoduodendal Blood supply from the inferior pancreaticoduodenal artery via the superior mesenteric artery can provide collateral blood supply to the head of the pancreas and the fi rst part of the duodenum in situations when the celiac trunk is occluded. Such anastomoses occur between the superior pancreaticoduodenal branches of the gastroduodenal artery (a derivative of the common hepatic branch of the celiac trunk) and the inferior pancreaticoduodenal.

A 3-year-old male is admitted to the pediatric clinic with a palpable mass in the right side of his scrotum, and a preliminary diagnosis is made of a congenital, indirect inguinal hernia. Which of the following is the most likely cause of an indirect inguinal hernia in this patient? A. The deep ring opens into an intact processus vaginalis. B. Congenital hydrocele C. Ectopic testis D. Epispadias E. Rupture of the transversalis fascia

A. the deep ring opens into an intact processus vaginalis Congenital inguinal hernias occur when a large patency of the processus vaginalis remains so that a loop of intestine herniates into the inguinal canal.

A 45-year-old male is admitted to the hospital with jaundice. This patient has a long history of alcoholism. Radiographic studies reveal ascites, portal hypertension, and liver cirrhosis. Which of the following veins are likely to be responsible for the production of ascitic fluid? A. Direct portal vein tributaries B. Anastomosing vessels between parietal veins and veins of retroperitoneal intestine C. Paraumbilical veins D. Esophageal submucosal anastomoses with azygos tributaries E. Superior rectal left gastric and middle rectal veins

B. Anastomosing vessels between paritetal veins and veins of retroperitoneal intestine Veins of the body wall and veins of the retroperitoneal intestinal organs are interconnected by extensive, thin-walled, anastomosing vessels. These veins tend to leak watery fluid in portal hypertension. Ascites is the accumulation of such fluid in the peritoneal cavity from these collateral veins. Ascites can occur within the veins of the body wall or veins of the retroperitoneal organs.

A 22-year-old man is admitted to the emergency department with acute abdominal pain at his right lower quadrant. Radiographic and physical examinations provide evidence of acute appendicitis. An appendectomy is performed, beginning with an incision at McBurney's point. Through which of the following abdominal layers must the surgeon pass to reach the appendix through this incision? A. External abdominal oblique muscle, internal oblique muscle, transversalis fascia, parietal peritoneum B. Aponeurosis of the external abdominal oblique muscle, internal oblique muscle, transversus abdominis muscle, transversalis fascia, parietal peritoneum C. Aponeurosis of the external abdominal oblique muscle, internal oblique muscle, transversus abdominis muscle, parietal peritoneum D. Aponeurosis of the external abdominal oblique muscle, aponeurosis of internal oblique muscle, transversus abdominis muscle, transversalis fascia, parietal peritoneum E. Aponeurosis of the external abdominal oblique muscle, aponeurosis of internal oblique muscle, aponeurosis of transversus abdominis muscle, transversalis fascia, pari- etal peritoneum

B. Aponeurosis of the external abdominal oblique muscle, internal oblique muscle, transversus abdominis muscle, transversalis fascia, parietal peritoneum The incision and tissue separation at McBurney's point to reach the appendix will usually en- counter the aponeurosis of the external abdominal oblique muscle, internal oblique muscle, transversus abdominis muscle, transversalis fascia, and peritoneum.

A 37-year-old woman was admitted to the emergency department with high fever (39.5° C), nausea, and vomiting. Physical examination revealed increased abdominal pain in the paraumbilical region, rebound tenderness over McBurney's point, and a positive psoas test. Blood tests showed marked leukocytosis. Which of the following is the most likely diagnosis? A. Ectopic pregnancy B. Appendicitis C. Cholecystitis D. Kidney stone E. Perforation of the duodenum

B. Appendicitis Appendicitis is often characterized by acute inflammation and is indicated with both a positive psoas test and rebound pain over McBurney's point.

A 30-year-old female patient has complained of weakness and fatigability over the past 6 months. She has a 3-month acute history of severe hypertension that has not responded to antihypertensive medications. Fasting serum glucose concentration is 140 mg/dl. A CT scan of the abdomen shows a 6-cm mass in the adrenal gland affecting the secretory cells of the adrenal medulla. Which of the following structures is most likely releasing products into the bloodstream to produce the hypertension and other signs? A. Preganglionic sympathetic axons in thoracic splanchnic nerves B. Cells of neural crest origin that migrated to the adrenal medulla C. Preganglionic parasympathetic branches of the posterior vagal trunk D. Postganglionic parasympathetic branches of the left or right vagus nerves E. Postganglionic fibers from pelvic splanchnic nerves

B. Cells of neural crest origin that migrated to the adrenal medulla The mass leads to increased stimulation and secretions of the chromaffin cells of the adrenal medulla. These cells are modified postganglionic sympathetic neurons of neural crest origin, and the epinephrine (adrenaline) and norepinephrine (noradrenaline) released by these cells passes into the suprarenal (adrenal) veins.

A 35-year-old male is admitted to the hospital from the emergency department because of excruciating pain in the back and left shoulder. A CT scan reveals an abscess in the upper part of the left kidney, but no abnormality is detected in the shoulder region. The shoulder pain may be caused by the spread of the inflammation to which of the following neighboring structures? A. Descending colon B. Diaphragm C. Duodenum D. Liver E. Pancreas

B. Diaphragm The abscess may have spread to the diaphragm and be causing the referred shoulder pain. This is because the diaphragm lies in close proximity to the inferior poles of the kidneys. The diaphragm is innervated by the phrenic nerves, bilaterally, which descend to the diaphragm from spinal nerve levels C3, C4, and C5. It is probably at the spinal cord that the referral of pain occurs between the phrenic nerve and somatic afferents entering at those levels.

A 23-year-old woman is admitted with severe ab- dominal pain, nausea, and vomiting. History taking shows that the pain is acute and has been constant for 4 days. The pain began in the epigastric region and radi- ated bilaterally around the chest to just below the scapu- lae. Currently the pain is localized in the right hypo- chondrium. A CT scan examination reveals calcified stones in the gallbladder. Which of the following nerves is carrying the afferent fibers of the referred pain? A. Greater thoracic splanchnic nerves B. Dorsal primary rami of intercostal nerves C. Phrenic nerves D. Vagus nerves E. Pelvic splanchnic nerves

B. Greater thoracic splanchnic nerves The greater splanchnic nerve carries general visceral afferent fibers from abdominal organs and can be involved in the occurrence of referred pain.

A 41-year-old woman is admitted to the hospital with upper abdominal pain. A gastroscopic examination reveals multiple small ulcerations in the body of the stomach. Which of the following nerves transmits the sensation of pain from this region? A. Spinal nerves T5 to T12 B. Greater thoracic splanchnic nerves C. Lesser thoracic splanchnic nerves D. Lumbar splanchnic nerves E. Spinal nerves T12 to L2

B. Greater thoracic splanchnic nerves The greater thoracic splanchnic nerves arise from the levels of the T5 to T9 thoracic sympathetic ganglia and are responsible for carrying general visceral afferents from upper abdominal organs and, therefore, from the body of the stomach.

A 50-year-old female with a long history of heart- burn (self-treated with various over-the-counter medications) develops severe epigastric pain and is urgently admitted to the hospital. A gastroscopic examination reveals a small, perforated ulceration in the posterior wall of the stomach body. At surgery, with the patient in supine position, 150 ml of blood-tinged, frothy gray liquid is aspirated from the peritoneal cavity. Where in the peritoneal cavity would liquid most likely first collect when the patient is supine? A. Right subphrenic space B. Hepatorenal pouch (of Morison) C. Left paracolic gutter D. Vesicouterine pouch E. Rectouterine pouch (of Douglas)

B. Hepatorenal pouch The hepatorenal pouch (or recess or space) is situated between the liver and both the parietal peritoneum covering the right kidney and suprarenal gland. This recess is the lowest space in the peritoneal cavity in supine patients. Accumulation of fluid in the peritoneal cavity will ordinarily collect in this pouch.

A 32-year-old woman was admitted to the hospi- tal with a complaint of pain over her umbilicus. Radio- graphic examination revealed acute appendicitis. The appendix was removed successfully in an emergency appendectomy. One week postoperatively the patient complained of paresthesia of the skin over the pubic region and the anterior portion of her perineum. Which of the following nerves was most likely injured during the appendectomy? A. Genitofemoral B. Ilioinguinal C. Subcostal D. Iliohypogastric E. Spinal nerve T9

B. Ilioinguinal The ilioinguinal nerve, which arises from the L1 spinal nerve, innervates the skin on the medial aspect of the thigh, scrotum (or labia majora), and the mons pubis. It has been injured in this patient.

A 32-year-old male is admitted to the emergency department with severe esophageal reflux. Radiographic examination reveals that the patient has a hiatal hernia, and a surgical procedure is scheduled. Which of the following landmarks would be the most useful to distinguish between sliding and paraesophageal hiatal hernias? A. Sliding hernias possess a normal gastroesoph- ageal junction. B. In sliding hernias the gastroesophageal junc- tion is displaced. C. Paraesophageal hernias have a displaced gas- troesophageal junction. D. In paraesophageal hernias the antrum moves into the stomach corpus. E. In paraesophageal hernias the antrum and the cardia move into the body of the stomach.

B. In sliding hernias the gastroesophageal junction is displaced. In sliding hernias the gastroesophageal junction is displaced. Diaphragmatic hernias of the esophagus can be characterized by analyzing the gastroesophageal junction. In sliding hernias the gastroesophageal junction is displaced anteriorly into the mediastinum. The paraesophageal hernia is generally characterized by herniation of the stomach into the mediastinum; however, the gastroesophageal junction remains fixed. In paraesophageal hernias the fundus herniates into the stomach, but the antrum does not.

A 61-year-old woman had been scheduled for a cholecystectomy. During the operation the scissors of the surgical resident accidentally entered the tissues immediately posterior to the epiploic (omental) foramen (its posterior boundary). The surgical field was filled immediately by profuse bleeding. Which of the following vessels was the most likely source of bleeding? A. Aorta B. Inferior vena cava C. Portal vein D. Right renal artery E. Superior mesenteric vein

B. Inferior vena cava The omental (epiploic) foramen (of Winslow) is the only natural opening between the lesser and greater sacs of the peritoneal cavity. It is bounded superiorly by the visceral peritoneum (liver capsule of Glisson) on the caudate lobe of the liver, inferiorly by the peritoneum on the first part of the duodenum, anteriorly by the free edge of the hepatoduodenal ligament, and posteriorly by the parietal peritoneum covering the inferior vena cava. Therefore, the inferior vena cava would be the most likely source of bleeding.

A 34-year-old man is undergoing an emergency appendectomy. After the appendectomy has been per- formed successfully, the patient undergoes an explor- atory laparoscopy. Which of the following anatomic features are the most useful to distinguish the jejunum from the ileum? A. Jejunum has thinner walls compared with the ileum. B. Jejunum has less mesenteric fat compared with the ileum. C. Jejunum has more numerous vascular arcades compared with the ileum. D. Jejunum has more numerous lymphatic folli- cles beneath the mucosa compared with the ileum. E. Jejunum has fewer villi compared with the ileum.

B. Jejunum has less mesenteric fat compared with the ileum. The jejunum makes up the proximal two fifths of the small intestine. There are several ways in which the ileum and jejunum differ. During surgery the easiest way to distinguish the two based on appearance is the relative amount of mesenteric fat. The jejunum has less mesenteric fat than the ileum.

A 57-year-old man is admitted to the emergency department with pain in his left flank and testicles. Laboratory tests indicate hematuria and anemia. A CT scan examination provides evidence that blood flow in the left renal vein is being occluded where it crosses anterior to the aorta. Which of the following is the most likely cause of the testicular pain? A. Compression of the testicular artery B. Occlusion of flow of blood in the testicular vein C. Compression of the afferent fibers in the lumbar splanchnic nerves D. Compression of the sympathetic fibers in the preaortic plexus E. Compression of the posterior vagus nerve

B. Occlusion of flow of blood in the testicular vein Blood flow would be impeded or greatly reduced in the left testicular vein because of the occlusion of the left renal vein into which the left testicular vein drains. This would result in pain as the testicular venous vessels become swollen.

A 56-year-old male is admitted to the hospital with severe abdominal pain. The patient has a history of "irritable bowel syndrome" affecting his rectum. Which of the following nerves will most likely be responsible for the transmission of pain in this case? A. Lumbar sympathetic chains B. Pelvic splanchnic nerves C. Pudendal nerves D. Sacral sympathetic chains E. Vagus nerves

B. Pelvic splanchnic nerves The visceral afferent innervation of the rectum is transmitted by way of the pelvic splanchnic nerves, which also provide the parasympathetic supply to this organ.

A major vessel appears to be nearly occluded in a 42-year-old male patient diagnosed with ductular adenocarcinoma. A CT scan examination has clearly demonstrated the tumor is at the neck of the pancreas. Which of the following vessels would be the most likely to be obstructed? A. Inferior mesenteric vein B. Portal vein C. Superior mesenteric artery D. Posterior superior pancreaticoduodenal artery E. Greater pancreatic artery

B. Portal vein The portal vein is the most likely structure to be occluded by a large tumor at the neck of the pancreas. The pancreas is drained via the splenic vein and empties into the portal vein. It directly enters the liver through the portal vein.

During a scheduled laparoscopic cholecystectomy in a 47-year-old female patient, the resident accidentally clamped the hepatoduodenal ligament instead of the cystic artery. Which of the following vessels would most likely be occluded in this iatrogenic injury? A. Superior mesenteric artery B. Proper hepatic artery C. Splenic artery D. Common hepatic artery E. Inferior vena cava

B. Proper hepatic artery The proper hepatic artery is the only artery typically within the hepatoduodenal ligament and therefore would be occluded. This artery lies within the right anterior free margin of the omental (or epiploic) foramen (of Winslow).

A 55-year-old man is admitted to the hospital with nausea, vomiting, and hematuria. A CT scan examination reveals a neoplasm in the posterior surface of the inferior pole of the left kidney that has invaded through the renal pelvis, renal capsule, ureter, and fat. To which of the following regions will pain most likely be referred? A. Skin of the anterior and lateral thighs and femoral triangle B. Skin over the gluteal region, pubis, medial thigh, and scrotal areas C. Skin over the medial, anterior, and lateral side of the thigh D. Skin over the pubis and umbilicus E. Skin over the pubis, umbilicus, and posterior abdominal wall muscles

B. Skin over the gluteal region, pubis, medial thigh, and scrotal areas Visceral pain from the kidneys and the ureter at the point of the neoplasm is mediated via T11 and T12 spinal cord levels. Therefore, pain is referred to these dermatomes leading to pain in the upper gluteal, pubic, medial thigh, scrotal, and labial areas (from subcostal and iliohypogastric nerves, in particular).

A 47-year-old female is admitted to the hospital with jaundice and epigastric pain that migrates toward the patient's right side and posteriorly toward the scapula. Radiographic examination reveals cholecystitis with a large gallstone. Which of the following is the most likely site for a gallstone to lodge? A. Common bile duct B. Hepatopancreatic ampulla C. Left hepatic duct D. Pancreatic duct E. Right hepatic duct

B. The hepatopancreatic ampulla The hepatopancreatic ampulla is also known as the ampulla of Vater and is located at the junction of the pancreatic duct and common bile duct. It is the narrowest part of the ductal system.

A 36-year-old man was brought to the emergency department with a bullet wound to the abdomen. The bullet penetrated the anterior abdominal wall superior to the umbilicus. If the bullet passed directly posterior in the midline, which of the following structures was most likely to have been struck first by the bullet? A. Abdominal aorta B. Transverse colon C. Stomach D. Gallbladder E. Pancreas

B. Transverse colon The bullet would probably first penetrate the transverse colon because it is the most superficial structure located slightly superior to the umbilicus.

A 15-year-old female is brought to the hospital with fever, nausea, and diffuse paraumbilical pain, which later becomes localized in the lower right quadrant. An appendectomy procedure is begun with an incision at McBurney's point. Which of the following landmarks best describes McBurney's point? A. The midpoint of the inguinal ligament in line with the right nipple B. Two thirds of the distance from the umbilicus to the anterior inferior iliac spine C. A line that intersects the upper one third of the inguinal ligament D. A line that intersects the lower third of the inguinal ligament, about 2 cm from the pubic tubercle E. One third of the distance from the anterior superior iliac spine to the umbilicus

B. Two thirds of the distance from the umbilicus to the anterior inferior iliac spine McBurney's point usually corresponds to the location of the base of the appendix where it attaches to the cecum. It is found on the right side of the abdomen, about two thirds of the distance from the umbilicus to the anterior superior iliac spine.

A 43-year-old man is admitted to the emergency department with complaints of intense abdominal pain. Radiographic examination reveals a right subphrenic abscess that extends to the midline. Which of the following structures would most likely be in a position to retard the spread of the abscess across the midline? A. Round ligament B. Falciform ligament C. Coronary ligament D. Hepatoduodenal ligament E. Gastroduodenal ligament

B. falciform ligament The falciform ligament separates the subphrenic spaces into right and left recesses and extends between the liver and the anterior abdominal wall. Because of its location and attachments, it would serve to stop the spread of such an abscess from one side to the other.

A 42-year-old male patient with jaundice is admitted to the hospital with severe pain that radiates to his back. A CT scan examination reveals a tumor at the neck of the pancreas. Biopsy reveals a ductular adenocarcinoma. Which of the following structures will first receive metastatic cells? A. Stomach B. Spleen C. Duodenum D. Liver E. Vertebral column

B. liver The liver would be the first structure to receive these metastatic cells because they would flow through the portal venous system from the pancreas to the liver.

A 70-year-old man is admitted to the emergency department with severe diarrhea. An arteriogram reveals 90% blockage at the origin of the inferior mesenteric artery from the aorta. Which of the following arteries would most likely provide collateral supply to the descending colon? A. Left gastroepiploic artery B. Middle colic artery C. Sigmoid artery D. Splenic artery E. Superior rectal artery

B. middle colic artery The middle colic artery can provide collateral supply to the descending colon when the inferior mesenteric artery is blocked or ligated. It is one of the first branches of the superior mesenteric artery and supplies the transverse colon. It provides collateral blood supply both to the ascending colon and descending colon by anastomosing with the right colic branch of the superior mesenteric artery and with the left colic artery, a branch from the inferior mesenteric artery.

A 45-year-old female is admitted to the hospital with symptoms of an upper bowel obstruction. Upon CT examination it is found that the third (transverse) portion of the duodenum is being compressed by a large vessel. Which of the following vessels will most likely be causing the compression? A. Inferior mesenteric artery B. Superior mesenteric artery C. Inferior mesenteric vein D. Portal vein E. Splenic vein

B. superior mesenteric artery The superior mesenteric artery arises from the aorta, behind the neck of the pancreas, and descends across the uncinate process of the pancreas and the third part of the duodenum before it enters the root of the mesentery behind the transverse colon. It can compress the third part of the duodenum.

A 32-year-old woman is admitted to the hospital with cramping abdominal pain around her umbilicus and vomiting for the previous 2 days. Radiographic studies indicate numerous stones in the gallbladder and air accumulation in the gallbladder and biliary tree. At which of the following places will an obstructive stone most likely be found? A. Jejunum B. Terminal ileum C. Common bile duct D. Duodenum E. Hepatic duct

B. terminal ileum Pain in the umbilical region can be indicative of referred pain from the large intestine. Gallstones can ulcerate through the wall of the fundus of the gallbladder and into the transverse colon, or through the wall of the body of the gallbladder into the duodenum. The stone would then most likely be entrapped at the ileocecal junction, possibly leading to an intestinal obstruction. This could lead predictably to the pain, cramping, and vomiting experienced by the patient.

A 36-year-old woman is admitted to the hospital for the imminent birth of her baby. The decision is made to perform an emergency cesarean section. A Pfannenstiel incision is used to reach the uterus by making a transverse incision through the external sheath of the rectus muscles, about 2 cm above the pubic bones. It follows natural folds of the skin and curves superior to the mons pubis. Which of the following nerves is most at risk when this incision is made? A. T10 B. T11 C. Iliohypogastric D. Ilioinguinal E. Lateral femoral cutaneous

C. Iliohypogastric The anterior cutaneous branch of the iliohypogastric nerve is responsible for the innervation of the skin above the mons pubis. This nerve arises from the T12 and L1 spinal nerves and runs transversely around the abdominal wall and over the lowest portion of the rectus sheath. It is the first cutaneous nerve situated superior to the mons pubis.

A 32-year-old male is admitted to the emergency department with groin pain. Examination reveals that the patient has an indirect inguinal hernia. Which of the following nerves is compressed by the herniating struc- ture in the inguinal canal to give the patient pain? A. Iliohypogastric B. Lateral femoral cutaneous C. Ilioinguinal D. Subcostal E. Pudendal

C. Ilioinguinal An indirect inguinal hernia occurs when a loop of bowel enters the spermatic cord through the deep inguinal ring (lateral to the inferior epigastric vessels). The ilioinguinal nerve runs with the spermatic cord to innervate the anterior portion of the scrotum and proximal parts of the genitals and could readily be compressed during an indirect inguinal hernia.

A 21-year-old woman is admitted to the hospital with a complaint of severe pain radiating from her lower back toward and above the pubic symphysis. Ultrasound examination reveals that a kidney stone is partially obstructing her right ureter. Which of the following nerves is most likely responsible for conducting the sensation of pain? A. Subcostal B. Iliohypogastric C. Ilioinguinal D. Lateral femoral cutaneous E. Obturator

C. Ilioinguinal The ureter is innervated by sympathetic and parasympathetic fibers in the ureteric plexus. General visceral afferent fibers in the ureteric plexus follow sympathetic fibers from spinal cord levels T11 to L2; therefore, pain from these fibers will be referred to nerves at these levels.

A 21-year-old football player is admitted to the emergency department with intense back pain. Physical examination shows that his left lower back is bruised and swollen. He complains of sharp pain during respiration. A radiograph reveals a fracture of the eleventh rib on the left side. Which of the following organs would be the most likely to sustain injury at this site? A. Spleen B. Lung C. Kidney D. Liver E. Pancreas

C. Kidney The kidney lies at the twelfth rib, and problems with pain associated with respiratory processes would result from the ribs being injured.

A 48-year-old woman visited the outpatient clinic with a complaint of lower left quadrant pain that had persisted for the previous 3 months. Laboratory examinations revealed that the patient had blood in her stools. A colonoscopy gave evidence of diverticulosis that had been affecting the distal part of the descending colon. To which of the following dermatomes would pain have most likely been referred? A. T5 to T9 B. T10 to L1 C. L1, L2 D. L1 to L4 E. T10 to L2

C. L1, L2 The descending colon receives its visceral sensory supply for pain from spinal segments L1 and L2. Injury to the descending colon can cause referral of pain to the corresponding dermatomes.

A 55-year-old man was admitted to the hospital with severe abdominal pain. Gastroscopy and CT scan examinations revealed a perforating ulcer in the posterior wall of the stomach. Where would peritonitis most likely develop initially? A. Right subhepatic space B. Hepatorenal space (of Morison) C. Omental bursa (lesser sac) D. Right subphrenic space E. Greater sac

C. Omental bursa The omental bursa is located directly posterior to the stomach and therefore would be the most likely space to develop peritonitis initially.

A 68-year-old woman is admitted to the hospital with severe pain radiating from her lower back toward her pubic symphysis. Ultrasound examination reveals that a renal calculus (kidney stone) is partially obstructing her right ureter. At which of the following locations is the calculus most likely to lodge? A. Major calyx B. Minor calyx C. Pelvic brim D. Midportion of the ureter E. Between the pelvic brim and the uterine cervix

C. Pelvic brim The ureter is normally constricted to some degree as it crosses the pelvic brim from major to minor pelvis.

A 62-year-old male internist is admitted to the emergency department with a complaint of severe chest pain. Physical examination reveals acute myocar- dial infarction. After the patient is stabilized, angiogra- phy is performed and the ejection fraction of the left ventricle is shown to be reduced to 30% of normal values. A cardiac pacemaker is placed to prevent fatal arrhythmias ( Fig. 2-4 ). What is the location of the tip of the pacemaker? A. Right atrium B. Left atrium C. Right ventricle D. Left ventricle E. Superior vena cava

C. Right atrium Artificial pacemakers are commonly used to treat patients who have weak or failing heart conduction systems. The electrode or "tip" of the pacemaker is threaded through the subclavian vein to the superior vena cava into the right atrium and then the right ventricle where it is used to stimulate the Purkinje fibers to result in ventricular contraction.

A 42-year-old female is admitted to the hospital after a traumatic landing while skydiving. Radiographic examination reveals a ruptured spleen. An emergency splenectomy is performed. Which of the following peritoneal structures must be carefully manipulated to prevent intraperitoneal bleeding? A. Coronary ligament B. Gastrocolic ligament C. Splenorenal ligament D. Phrenocolic ligament E. Falciform ligament

C. Splenorenal ligament The splenorenal ligament is the attachment of the spleen to the left kidney and is the only ligament that contains the major branches of the splenic artery to the spleen and greater curvature of the stomach.

A 38-year-old male is examined in the outpatient clinic because of his complaint of mild abdominal pain of 2 years' duration. Upon examination, it is observed that the pain is dull and located principally in the left upper quadrant around the xiphoid process. An endoscopic examination reveals that the patient suffers from a gastric ulcer. At which of the following spinal nerve levels are the neuronal cell bodies located for the sensory fibers in such a case of gastric ulcer? A. T5, T6 B. T6 to T8 C. T7, T8 D. T9 to L1 E. T5 to T9

C. T7, T8 The spinal cord levels containing the soma of the sensory fibers transporting the sensation of pain are more than likely at the level of T7 and T8. This is because the xiphoid process is at these dermatome levels for somatic sensations of pain, and these same spinal nerves receive visceral afferents from the stomach.

A 27-year-old male billiards player received a small-caliber bullet wound to the chest in the region of the third intercostal space, several centimeters to the left of the sternum. The patient is admitted to the emer- gency department and a preliminary notation of "Beck's triad" is entered on the patient's chart. Which of the following are features of this triad? A. There was injury to the left pulmonary artery, left primary bronchus, and esophagus. B. The patient has bleeding into the pleural cav- ity, a collapsed lung, and mediastinal shift to the right side of the thorax. C. The patient has a small, quiet heart; de- creased pulse pressure; and increased central venous pressure. D. The young man is suffering from marked dia- stolic emptying, dyspnea, and dilation of the aortic arch. E. The left lung has collapsed, there is paradoxi- cal respiration, and there is a mediastinal shift of the heart and trachea to the left.

C. The patient has a small, quiet heart, decreased pulse pressure, and increased central venous pressure The patient is suffering from cardiac tamponade, that is, filling of the pericardial cavity with fluid. The classic signs of this tamponade are referred to as "Beck's triad." This trio, by definition, includes a small heart, from compression of the heart by the fluid-filled pericardial sac, and a quiet heart because the tamponade muffles the cardiac sounds; decreased pulse pressure resulting from the reduced difference between systolic and diastolic pressure because the tamponade restricts the ability of the heart to fill in diastole; and increased central venous pressure because venous blood cannot enter the compressed heart.

A 44-year-old man is admitted to the emergency department with excessive vomiting and dehydration. Radiographic images demonstrate that part of the bowel is being compressed between the abdominal aorta and the superior mesenteric artery. Which of the following intestinal structures is most likely being compressed? A. Second part of duodenum B. Transverse colon C. Third part of duodenum D. First part of duodenum E. Jejunum

C. Third part of the duodenum The third part of the duodenum takes a path situated anterior to the abdominal aorta and inferior to the superior mesenteric artery (a major ventral branch of the abdominal aorta). Because the third part of the duodenum lies in the angle between ("sandwiched") these two structures, constrictions of this portion of the duodenum can occur readily.

A 64-year-old man is admitted to the hospital with intense abdominal pain from a pancreatic tumor. A neurectomy is performed to interrupt the neural fibers supplying the pancreas. Which of the following neural fibers would be the most likely objective of the neurectomy? A. Sympathetic preganglionic B. Sympathetic postganglionic C. Visceral afferent D. Postganglionic preganglionic E. Postganglionic postganglionic

C. Visceral afferent Visceral afferents are nerve fibers that transmit the sensation of visceral pain. The target of the neurectomy would be to eradicate visceral pain. One would not want to interfere with sympathetic and parasympathetic nerves, as these provide motor innervation to viscera.

A 58-year-old man was admitted to the hospital with complaints of pain in the right upper quadrant and jaundice. Ultrasound examination reveals numerous large gallstones in his gallbladder. Which of the following nerves would transmit the pain of cholecystitis? A. The right vagus nerve, with referral to the inferior angle of the scapula B. Afferent fibers in spinal nerves T1 to T4 C. Visceral afferent fibers in the greater thoracic splanchnic nerve, with referral to the derma- tomes from T6 to T8 D. Sympathetic T10 to T12 portions of greater thoracic splanchnic nerve via celiac ganglion and celiac plexus E. Afferent fibers of dorsal primary rami of spinal nerves T6 to T8, with referral to the epigastric region

C. Visceral afferent fibers in the greater thoracic splanchnic nerve with referal to the dermatomes from T6 to T8 Cholecystitis is an inflammation of the gallbladder due to increased concentration of bile or obstruction of the cystic duct by gallstones. Pain is ultimately felt in the right hypochondriac region, which corresponds to the T6 to T8 dermatomes.

A 58-year-old male alcoholic is admitted to the hospital after vomiting dark red blood (hematemesis). Endoscopy reveals ruptured esophageal varices, resulting from portal hypertension. Which of the following venous tributaries to the portal system anastomoses with caval veins to cause the varices? A. Splenic B. Left gastroomental C. Left gastric D. Left hepatic E. Right gastric

C. left gastric The left gastric vein carries blood from the stomach to the portal vein. At the esophageal gastric junction the left gastric vein (portal system) anastomoses with esophageal veins (caval system). High blood pressure in the portal system causes high pressure in this anastomosis, causing the ruptured esophageal varices.

A 67-year-old man has severe cirrhosis of the liver. He most likely has enlarged anastomoses between which of the following pairs of veins? A. Inferior phrenic and superior phrenic B. Left colic and middle colic C. Left gastric and esophageal D. Lumbar and renal E. Sigmoid and superior rectal

C. left gastric and esophageal Cirrhosis of the liver would lead to inability of the portal system to accommodate blood fl ow. Blood backs up toward systemic circulation, draining to the inferior vena cava, with pooling at areas of portal-caval anastomoses. The left gastric vein (portal) meets the esophageal vein (caval) and enlarges or expands in instances of cirrhosis.

A 55-year-old male is admitted to the hospital because of severe weight loss over the preceding 6-month period of time. Radiographic examination and other tests provide evidence that a tumor is causing portal hypertension. Laboratory studies reveal that the patient has fatty stool, malnutrition, and liver hypoxia. At which of the following locations is the tumor most likely located? A. Right lobe of the liver B. Left lobe of the liver C. Porta hepatis D. Falciform ligament E. Hepatogastric ligament

C. porta hepatis The porta hepatis (transverse fissure of liver) transmits the proper hepatic artery, portal vein, common hepatic duct, autonomic nerves, and lymph vessels. A tumor in this region would be most detrimental because of its abundance of vessels and lymphatics that could lead to all of these symptoms when they are compromised functionally.

A 42-year-old woman is admitted to the hospital with an acutely painful abdomen. Radiographic examination reveals penetration of a posterior duodenal ulcer resulting in intraabdominal bleeding. Which of the following arteries is most commonly eroded by this type of ulcer? A. Gastroduodenal artery B. Superior mesenteric C. Posterior superior pancreaticoduodenal D. Posterior inferior pancreaticoduodenal E. Right gastric

C. posterior superior pancreaticoduodenal Perforation of a posterior duodenal ulcer most commonly damages the posterior superior pancreaticoduodenal artery. This artery branches from the inferior aspect of the gastroduodenal artery.

A 54-year-old man is admitted to the hospital with vomiting and severe weight loss. Physical examination reveals that the umbilical and epigastric regions are tender and painful. A CT scan examination reveals a massive tumor originating from the third part of the duodenum. Which of the following structures is more likely to be compressed or invaded by the tumor? A. Common bile duct B. Portal vein C. Superior mesenteric artery D. Gastroduodenal artery E. Posterior superior pancreaticoduodenal artery

C. superior mesenteric artery The superior mesenteric artery arises from the aorta, behind the neck of the pancreas, at the level of L1 within the abdominal cavity, and traverses inferiorly across the anterior surface of the third part of the duodenum. As it crosses the uncinate process of the pancreas and duodenum, this artery could readily be affected by a tumor in the immediate area.

A 45-year-old woman is admitted to the emergency department with a complaint of severe abdominal pain. CT scan and MRI examinations reveal a tumor of the head of the pancreas involving the uncinate process. Which of the following vessels is most likely to be occluded? A. Common hepatic artery B. Cystic artery and vein C. Superior mesenteric artery D. Inferior mesenteric artery E. Portal vein

C. superior mesenteric artery The superior mesenteric artery arises from the aorta, deep to the neck of the pancreas, then crosses the uncinate process and third part of the duodenum. An uncinate tumor can cause compression of the superior mesenteric artery.

A 46-year-old man is admitted to the hospital with a rather large but painless mass on his right groin. During physical examination it is noted that the inguinal lymph nodes are hard and palpable. A lymph node biopsy reveals the presence of malignant cells. Which of the following locations would be the most likely primary source of carcinoma? A. Prostate B. Bladder C. Testis D. Anal canal E. Sigmoid colon

D. Anal Canal A malignancy of the anal canal would drain into the inguinal lymph nodes, specifically the superficial lymph nodes. The internal iliac lymph nodes receive drainage from the rectum, the uterus, the prostate gland, and the bladder.

An 80-year-old male patient is admitted to the hospital with hypertension. His history includes a notation that he has had a poor appetite for some time. During physical examination it is observed that his blood pressure is 175/95 mm Hg and that he has a marked pulsation in his epigastric region. Which of the following diagnoses will most likely explain the symptoms and signs? A. Hiatal hernia B. Splenomegaly C. Cirrhosis of the liver D. Aortic aneurysm E. Kidney stone

D. Aortic aneurysm The aortic aneurysm often occurs between L3 and L4, below the bifurcation of the aorta, resulting in significant increase in pressure, creating the marked abdominal pulsation.

A 19-year-old male is admitted to the hospital after a violent automobile collision. An MRI examination reveals that the spinal cord has been transected at the L4 cord level. Which of the following portions of the intestine will most predictably lose parasympathetic innervation from the central nervous system? A. Jejunum B. Ascending colon C. Ileum D. Descending colon E. Transverse colon

D. Descending colon Descending colon. Below the left colic flexure, innervation of the gastrointestinal tract is supplied by parasympathetic fibers of the pelvic splanchnic nerves.

The 22-year-old female patient cried out in agony from the pain of her burst appendix. Which of the following structures contain the neuronal cell bodies of the pain fi bers from the appendix? A. Sympathetic chain ganglia B. Celiac ganglion C. Lateral horn of the spinal cord D. Dorsal root ganglia of spinal nerves T8 to T10 E. Dorsal root ganglia of spinal nerves L2 to L4

D. Dorsal root ganglia of spinal nerves T8 to T10 The dorsal root ganglia contain all cell bodies of sensory neurons from the body wall and limbs. Afferent fibers from the appendix travel in T8 to T10. The sympathetic chain contains postganglionic sympathetic cell bodies that are targeted to smooth muscle and glands of the viscera and heart muscle.

A 55-year-old male had been unsuccessfully treated for alcoholism for 3 years. He was admitted to the hospital for emergency medical treatment for severe portal hypertension. Which of the following is a feature of the development of severe portal hypertension? A. Esophageal varices—from increased pressure in the right gastric vein B. Ascites—from effusion of fl uid from the infe- rior mesenteric vein C. Internal hemorrhoids—from increased pres- sure within the superior mesenteric vein and its tributaries D. Expansion of veins within the falciform liga- ment, which anastomose with veins of the umbilical region E. Recanalization and expansion of the vessels within the medial umbilical ligaments

D. Expansion of veins within the falciform ligament, which anastomose with veins of the umbilical region Caput medusae is an end-stage characteristic of liver cirrhosis. The snakelike appearance of veins on the body wall results from anastomoses between tiny veins that accompany the ligamentum teres (that is within the falciform ligament) with veins of the body wall. The umbilical veins are expanded, due to portal hypertension.

A 51-year-old woman is admitted to the hospital with an acutely painful abdomen. Radiographic examination reveals penetration of the fundic region of the stomach by an ulcer, resulting in intraabdominal bleeding. Which of the following arteries is the most likely source of the bleeding? A. Common hepatic artery B. Inferior phrenic artery C. Left gastroepiploic artery D. Short gastric artery E. Splenic artery

D. short gastric artery The most likely candidate for bleeding from the fundic region of the stomach in this case would be either the short gastric or dorsal gastric branches of the splenic artery. The short gastric arteries pass from the area of the splenic hilum to the fundus, supplying anterior and posterior branches to this part of the stomach.

A 35-year-old man is admitted to the hospital with a small-caliber bullet wound to the left upper quadrant of the abdomen. Radiographic examination reveals profuse intraperitoneal bleeding. An emergency laparotomy is performed, and the source of bleeding appears to be a vessel within the lesser sac. Which of the following ligaments would most likely be transected to gain adequate entry to the lesser sac? A. Coronary B. Gastrosplenic C. Splenorenal D. Gastrocolic E. Hepatoduodenal

D. Gastrocolic The lowest point of the lesser sac occurs at the intersection of the gastrocolic ligament and transverse colon. Bleeding would travel to the lowest point of the lesser sac. Access to this area would require entry through the gastrocolic ligament, which extends from the greater curvature of the stomach to the transverse colon.

Two days after an appendectomy on a 45-year-old male patient, the patient has developed an elevated temperature (39° C), is hypotensive, and complains of abdominal pain. An exploratory laparotomy reveals large amounts of blood in the peritoneal cavity due to an injury to a vessel that occurred during the appendectomy. Which of the following vessels must be ligated to stop the bleeding? A. Right colic artery B. Right colic artery and superior rectal artery C. Superior mesenteric artery D. Ileocolic artery E. Ileocolic artery and middle colic artery

D. Ileocolic artery The ileocolic artery is the only artery listed that directly supplies the appendix.

A 39-year-old woman is admitted to the hospital with pain radiating to her inguinal region. Radiographic and physical examination reveal a herniation. Which of the following is the most common type of hernia in a female patient? A. Femoral hernia B. Umbilical hernia C. Direct inguinal hernia D. Indirect inguinal hernia E. Epigastric hernia

D. Indirect inguinal hernia Indirect inguinal hernia is the most common groin hernia in females.

A 55-year-old man is admitted to the hospital for his annual checkup. An ultrasound examination reveals a tumor that has incorporated the right common iliac artery and compressed the vein that lies posterior to it. Doppler ultrasound studies give evidence of the development of a deep venous thrombosis that could block venous return from the left lower limb, causing ischemia and pain. Which of the following vessels is most likely to be involved in the production of the deep venous thrombosis? A. Inferior vena cava B. Right renal vein C. Left testicular vein D. Left common iliac vein E. Right common iliac vein

D. Left common iliac vein The left common iliac vein lies posterior to the right common iliac artery. Compression of the vein in this location is a frequent cause of deep venous thrombosis of the left lower limb; that is, the venous drainage of the lower limb is obstructed. This can cause extreme pain, together with ischemia of the limb that, in some untreated cases, can lead to amputation of the limb or death.

A 60-year-old man is admitted to the emergency department with severe abdominal pain. Physical examination reveals guarding and rigidity in the abdominal wall. An abdominal CT scan shows a thrombus in an intestinal artery supplying the ileum. Which of the following layers of peritoneum will have to be entered by the surgeon to access the affected vessel? A. Parietal peritoneum and the greater omentum B. Greater and lesser omentum C. Lesser omentum and the gastrosplenic ligament D. Parietal peritoneum and the mesentery E. Greater omentum and the transverse mesocolon

D. Parietal peritoneum and mesentery Parietal peritoneum and "the mesentery." The parietal peritoneum lines the abdominal wall, whereas the visceral peritoneum is in intimate contact with organs. The greater omentum extends from the greater curvature of the stomach and covers the midgut. Access to the ileum would require penetration of the parietal peritoneum to enter the peritoneal cavity and interruption of the visceral peritoneum of "the mesentery" covering the thrombosed vessel.

A 48-year-old woman is admitted to the emergency department with a complaint of severe abdominal pain. Radiographic examination reveals advanced carcinoma of the head of the pancreas. A celiac plexus block is performed to relieve her pain. Which of the following best describes the nerve structures that are most likely to be present in the celiac ganglion? A. Preganglionic parasympathetic and somatic motor fi bers B. Postganglionic parasympathetic and visceral afferent fi bers C. Postganglionic sympathetic and visceral affer- ent fibers D. Pre- and postganglionic sympathetic, pregan- glionic parasympathetic, and visceral afferent fi bers E. Preganglionic sympathetic, preganglionic parasympathetic, and visceral afferent fi bers

D. Pre- and postganglionic sympathetic, preganglionic parasympathetic, and visceral afferent fibers Preganglionic and postganglionic sympathetics, preganglionic parasympathetic, and visceral afferent fibers are present within the celiac ganglion. The cell bodies of postganglionic sympathetic fibers are contained within the celiac ganglion and their axons pass to upper abdominal organs.

A 45-year-old woman is admitted to the hospital after her automobile left the highway in a rainstorm and hit a tree. On radiographic examination, it is observed that she has suffered fractures of the ninth and tenth rib on her left side and that she has intraabdominal bleeding. Physical examination reveals hypovolemic shock and progressive hypotension. Which of the following organs is most likely injured to result in these clinical signs? A. Liver B. Pancreas C. Left kidney D. Spleen E. Ileum

D. Spleen The spleen is a large lymphatic organ that rests against the diaphragm and ribs 9, 10, and 11 in the left hypochondriac area. A laceration of the organ is often associated with severe blood loss and shock.

A 48-year-old man has had three episodes of upper gastrointestinal bleeding from esophageal varices. He has a history of chronic alcoholism but has recently been rehabilitated. Further evaluation shows ascites and splenomegaly. Which of the following surgical venous anastomoses is most commonly used to relieve these symptoms and signs before a liver transplant is attempted? A. Left gastric to splenic vein B. Right gastric to left gastric vein C. Right renal to right gonadal vein D. Splenic to left renal vein E. Superior mesenteric to inferior mesenteric vein

D. Splenic to left renal vein Surgical anastomoses to alleviate symptoms of portal hypertension are rooted in the premise that connection of a large portal vein to a large systemic vein allows for collateral drainage of the portal system. The splenic vein, a component of the portal venous system, and the left renal vein, a component of the caval-systemic venous system, are ideally located to allow for a low-resistance, easily performed anastomosis.

A 52-year-old male presents to the emergency department complaining of persistent severe right upper quadrant pain for the past 2 hours. During that period of time he felt nauseated, was sweating profusely, and also experienced pain in the posterior aspect of his right shoulder. The pain began shortly after a lunch consisting of "fast food." Ultrasound examination reveals multiple stones in an inflamed gallbladder with a normal bile duct. Which of the following spinal nerve segments are involved in the shoulder pain, associated with cholecystitis? A. C3 to C5 B. C5 to C8 C. T1 to T4 D. T5 to T9 E. T10, T11

D. T5 to T9 Referred pain from cholecystitis is generally referred to the region of the inferior angle of the right scapula. These fibers are generally from T5 to T9. These sensory fibers for pain are stimulated by the gallbladder inflammation because of the proximity of the adjacent structures.

A 52-year-old male with a history of smoking and hypercholesterolemia is diagnosed with severe athero- sclerosis affecting the arteries of his body. Laboratory examination reveals extremely low sperm count. Which of the following arteries is most likely occluded? A. External iliac B. Inferior epigastric C. Umbilical D. Testicular E. Deep circumflex iliac

D. Testicular The testicular artery originates from the abdominal aorta and travels with the spermatic cord, leading to the testes in the male.

Exploratory laparoscopy was performed on a 34-year-old male, following a successful emergency appendectomy. Which of the following anatomic relationships would be seen clearly, without dissection, when the surgeon exposes the beginning of the jejunum? A. The second portion of the duodenum is related anteriorly to the hilum of the right kidney. B. The superior mesenteric artery and vein pass posterior to the third part of the duodenum. C. The portal vein crosses anterior to the neck of the pancreas. D. The second part of the duodenum is crossed anteriorly by the attachment of the transverse mesocolon. E. The third part of the duodenum is related anteriorly to the hilum of the left kidney.

D. The second part of the duodenum is crossed anteriorly by the attachment of the transverse mesocolon. The second part of the duodenum is crossed anteriorly by the transverse mesocolon, a relationship that can be seen when the beginning of the jejunum is exposed by lifting the transverse colon superiorly.

A 24-year-old woman is admitted to the hospital with lower abdominal pain. A CT examination reveals an abnormal mass occupying the left adnexa in the pelvis. During the surgical procedure the ureter and the structures immediately medial to the ureter are identified. Which of the following vascular structures crosses the ureter just lateral to the cervix of the uterus? A. Middle rectal artery B. Superior vesical artery C. Internal pudendal vein D. Uterine artery E. Gonadal vein

D. Uternine artery Vascular structures situated immediately medial to the ureter are often subject to ligation during surgical procedures. The ureter is crossed by the uterine artery an inch or so lateral to the cervix and must be identified and avoided, in ligating the uterine vessels.

A 45-year-old woman is admitted to the hospital with rectal bleeding. Physical examination, including a rectal examination, reveals an abnormal mass of tissue protruding below the pectinate line. Biopsy reveals the presence of an adenocarcinoma. Which of the following groups of lymph nodes would first receive lymph from the area of pathology? A. Internal iliac B. External iliac C. Middle rectal D. Superficial inguinal E. Deep inguinal

D. superficial inguinal Superficial inguinal nodes. The external anal sphincter is skeletal muscle of the anal canal. This suggests that the carcinoma is originating from the anal canal. The results of the biopsy support the finding that the carcinoma most likely occurred below the pectinate line where you would find squamous cells of the anal canal. The anal canal primarily drains to the superficial inguinal lymph nodes.

A 23-year-old man is admitted to the hospital with a bulge in his scrotum. Physical examination reveals an indirect inguinal hernia. During the open hernia repair the internal spermatic fascia is identified and reflected to expose the ductus deferens and testicular vessels. Which of the following provides the internal spermatic fascial layer of the spermatic cord? A. External abdominal oblique aponeurosis B. Internal abdominal oblique aponeurosis C. Transversus abdominis aponeurosis D. Transversalis fascia E. Processus vaginalis

D. transversalis fascia The transversalis fascial layer is the source of the internal spermatic fascia. The walls of the spermatic cord consist of three layers: external spermatic fascia, cremaster muscle, and the internal spermatic fascia.

During a laparoscopic cholecystectomy on a 61-year-old male, which of the following arteries must be clamped to remove the gallbladder safely? A. Common hepatic B. Proper hepatic C. Right hepatic D. Left hepatic E. Cystic

E. Cystic The cystic artery is the only artery listed that goes directly to the gallbladder. It is often a branch of the right hepatic artery and must be clamped before the gallbladder is cut free from its attachments.

A 45-year-old man is admitted to the hospital with a palpable and painful mass at his groin that is exacerbated when he stands erect or physically exerts himself. Physical examination indicates the probability of a direct inguinal hernia, a diagnosis that is confirmed laparoscopically. Which of the following is the most likely cause of this type of inguinal hernia? A. Defective transversalis fascia around the deep inguinal ring B. Defective peritoneum around the deep ingui- nal ring C. Defective aponeurosis of external abdominal oblique muscle D. Defective extraperitoneal connective tissue E. Defective aponeurosis of transversus abdominis muscle

E. Defective aponeurosis of transversus abdominis muscle The transversus abdominis aponeurosis and transversalis fascia form a significant portion of the posterior wall of the inguinal canal and the lower part of the inguinal triangle (of Hesselbach). Gradual weakness or attrition of tissues in the posterior wall provides the likelihood of egress of a direct inguinal hernia.

A 34-year-old patient had been diagnosed earlier in the week with Guillain-Barré syndrome. He is now in extreme respiratory distress. His thoracic wall con- tracts and relaxes violently, but there is little movement of the abdominal wall. The degenerative disease has obviously affected the muscle that is most responsible for increasing the vertical dimensions of the thoracic cavity (and pleural cavities). Which of the following is the most likely cause of his disease? A. Paralysis of his intercostal muscles and loss of the "bucket handle movement" of his ribs B. Generalized intercostal nerve paralysis that resulted in loss of the "pump handle movement" of his ribs C. Paralysis of his medial and lateral pectoral nerves, interrupting the function of his pectoralis major muscles, an important accessory muscle of respiration D. Paralysis of his sternocleidomastoid muscles E. Degeneration of the myelin of his phrenic nerves

E. Degeneration of the myelin of his phrenic nerves Myelin degeneration of the phrenic nerves, as can occur in Guillain-Barré, results in loss of phrenic nerve function and paralysis of the diaphragm. Diaphragmatic paralysis is predictable with lack of movement of the abdominal wall in respiratory efforts.

A 15-year-old boy underwent an appendectomy procedure. Two weeks postoperatively the patient complains of numbness of the skin over the pubic region and anterior portion of his genitals. Which of the following nerves was most likely iatrogenically injured during the operation? A. Pudendal B. Genitofemoral C. Spinal nerve T10 D. Subcostal E. Ilioinguinal

E. Ilioinguinal The ilioinguinal nerve is a terminal branch of spinal nerve L1. It innervates the skin overlying the iliac crest; the anterior portion of the urogenital region; and the upper, inner thigh. Its pathway takes it below the typical site of McBurney's point, but it can be injured with extension of an appendectomy incision.

The decision is made by emergency department surgeons to perform an exploratory laparotomy on a 32-year-old female with severe abdominal pain. Where would the incision most likely be made to separate the left and right rectus sheaths? A. Midaxillary line B. Arcuate line C. Semilunar line D. Tendinous intersection E. Linea alba

E. Linea alba The linea alba is formed by the intersection of aponeurotic tissues between the right and left rectus abdominal muscles. It contains the aponeuroses of the abdominal muscles and is located at the midline of the body.

A 47-year-old male patient had undergone bilateral vagotomy, with division of both vagus trunks at the esophageal hiatus, to relieve his chronic difficulty with peptic ulcers. Which of the following conditions will most likely occur? A. Parasympathetic supply to the descending colon is lost. B. The patient would no longer have contraction of the urinary bladder. C. The patient would become impotent. D. The patient would be sterile because of paralysis of the ductus deferens and ejaculatory duct. E. Parasympathetic supply to the ascending co- lon would be reduced or absent.

E. Parasympathetic sypply to the ascending colon would be reduced or absent Interruption of both vagus nerves would deprive the abdominal viscera of parasympathetic supply, that is, to the level of the splenic flexure of the colon. Distal to the splenic flexure, the colon receives parasympathetic nerves.

A 48-year-old man is admitted to the hospital with severe abdominal pain. Radiographic examination reveals a tumor in the tail of the pancreas. A diagnostic arteriogram shows that the tumor has compromised the blood supply to another organ. Which of the following organs is most likely to have its blood supply compromised by this tumor? A. Duodenum B. Gallbladder C. Kidney D. Liver E. Spleen

E. Spleen The splenic artery lies adjacent to the superior border of the pancreas. The organ it principally supplies is the spleen, which is located at the termination of the pancreatic tail. Blood supply to the spleen can therefore be affected in the event of a tumor in the tail of the pancreas.

A 29-year-old man is admitted to the hospital with great difficulty breathing after an automobile accident. Radiographic examination reveals no fractured bones or mediastinal shift. During physical examination he has no signs of external injuries, but the dyspnea becomes progressively worse. Which of the following conditions would best describe this case? A. Flail chest with paradoxical respiration B. Emphysema C. Hemothorax D. Spontaneous pneumothorax E. Tension pneumothorax

E. Tension pneumothorax Tension pneumothorax is a progressive accumulation of air in the pleural cavity that is trapped during inspiration. The resulting increase of pressure diminishes the negative pressure required to maintain an inflated lung, resulting in a collapsed lung as seen on the radiograph.

A 40-year-old male with nine children is urged by his wife to have a vasectomy. During the operation the urologist separates the layers of the spermatic cord to expose the ductus deferens so that it can be ligated and cut. From what structure is the internal spermatic fascial covering derived? A. Internal oblique muscle B. Cremaster muscle C. External abdominal oblique muscle fascia D. Transversus abdominis aponeurosis E. Transversalis fascia

E. Transversalis fascia The innermost covering of the spermatic cord is the internal spermatic fascia. It originates from the transversalis fascia.

A 61-year-old man is admitted to the emergency department with abdominal pain and a 2-day history of vomiting. Physical examination reveals a colicky abdominal pain in the right lower quadrant, with abdominal distention. Upon auscultation, episodes of pain were associated with rushes, gurgling, and tinkling sounds. A CT scan examination reveals a mechanical obstruction of the bowel. Which of the following parts of the gastrointestinal tract is most likely obstructed? A. Hepatopancreatic ampulla of Vater B. Duodenal bulb C. Proximal ileum D. Pyloric sphincter E. Ileocecal junction

E. ileocecal junction A gallstone ileus occurs when a gallstone (cholelith) ulcerates through the wall of the body of the gallbladder and into the duodenum. In this case, gallstones became lodged in the ileocecal region. Obstruction in the ileocecal junction can produce pain that mimics appendicitis. However, bowel sounds will be exaggerated above the obstruction and absent distal to the obstruction. This obstruction would require surgical correction.

A 45-year-old male was admitted to the hospital with groin pain and a palpable mass just superior to the inguinal ligament. The patient was diagnosed with an inguinal hernia and a surgical repair was performed. During the operation the surgeon found a loop of intestine passing through the deep inguinal ring. Which of the following types of hernia was this? A. Direct inguinal B. Umbilical C. Femoral D. Lumbar E. Indirect inguinal

E. indirect inguinal Indirect hernias commonly result from herniation of the intestines through the deep inguinal ring.

A 57-year-old man is admitted to the emergency department with left flank pain. Blood tests indicate hematuria and anemia. A magnetic resonance scan reveals that blood flow in the left renal vein is being occluded by an arterial aneurysm where the vein crosses the aorta. The aneurysm is most likely located in which of the following arteries? A. Celiac B. Inferior mesenteric C. Left colic D. Middle colic E. Superior mesenteric

E. superior mesenteric The superior mesenteric artery lies just superior and anterior to the left renal vein as the vein passes to its termination in the inferior vena cava.

During the surgical repair of a perforated duodenal ulcer in a 47-year-old male patient, the gastroduodenal artery is ligated. A branch of which of the following arteries will continue to supply blood to the pancreas in this patient? A. Inferior mesenteric B. Left gastric C. Right gastric D. Proper hepatic E. Superior mesenteric

E. superior mesenteric The superior mesenteric artery will supply the pancreas if the gastroduodenal artery is ligated. It arises immediately inferior to the celiac trunk from the thoracic aorta. Its first branches are the anterior and posterior inferior pancreaticoduodenal arteries, which aid the superior pancreaticoduodenal arteries (which take origin from the gastroduodenal branch of the celiac trunk) in supplying the pancreas with oxygenated


Conjuntos de estudio relacionados

HESI Case Study: Medical/Surgical: Osteoporosis Kat Mitchell

View Set

Medical Terminology Chapter 4 Anticipation Guide

View Set

NCLEX RN EXAM PRACTICE QUESTIONS L/M

View Set

Life Insurance Chapter 2. General Wisconsin Insurance Laws

View Set